Exam 4 Shocks, MODS, SIRS Flashcards

1
Q

Which types of shock may cause reduced urinary output in a patient? [Select all that apply]

A. Septic shock
B. Hypovolemic shock
C. Neurogenic shock
D. Anaphylactic shock
E. Cariogenic shock.
A

A. Septic shock
B. Hypovolemic shock
E. Cariogenic shock.

Decreased urine output is a clinical manifestation of septic, hypovolemic, obstructive and cardiogenic shock. Neurogenic shock is associated with bladder dysfunction. Anaphylactic shock is associated with urinary incontinence.

How well did you know this?
1
Not at all
2
3
4
5
Perfectly
2
Q

Following coronary artery bypass graft surgery a patient has postoperative bleeding that requires returning to surgery to repair the leak. During surgery, the patient has a myocardial infarction (MI). After restoring the patient’s body temperature to normal, which patient assessment is the most important for planning nursing care?

A. Cardiac index (CI) 5 L/min/m2
B. Central venous pressure (CVP) 8 mm Hg
C. Mean arterial pressure (MAP) 86 mm Hg
D. Pulmonary artery pressure (PAP) 28/14 mm Hg

A

D. Pulmonary artery pressure (PAP) 28/14 mm Hg

Pulmonary hypertension as indicated by an elevated PAP indicates impaired forward flow of blood because of left ventricular dysfunction or hypoxemia. Both can be caused by the MI. The CI, CVP, and MAP readings are normal.

How well did you know this?
1
Not at all
2
3
4
5
Perfectly
3
Q

A patient in neurogenic shock is receiving phenylephrine. Which nursing actions are appropriate when caring for this patient?

A. Monitoring for signs of dyspnea and pulmonary edema
B. Monitoring for signs of hypokalemia and hyperglycemia
C. Monitoring for signs of reflex bradycardia and restlessness
D. Monitoring for signs of hypothyroidism and Addison’s disease

A

C. Monitoring for signs of reflex bradycardia and restlessness

Phenylephrine is α-adrenergic agonist and may cause bradycardia and restlessness due to central nervous system stimulation. β-adrenergic agonists such as epinephrine cause dyspnea and pulmonary edema. Phenylephrine does not increase the elimination of potassium levels or blood glucose levels. Therefore, the patient does not have risk of hypokalemia and hyperglycemia. Phenylephrine does not impair thyroid and adrenal gland functioning. Therefore, the nurse will not monitor for the signs of hypothyroidism and Addison’s disease.

How well did you know this?
1
Not at all
2
3
4
5
Perfectly
4
Q

When managing a patient with shock, which appropriate actions should the nurse take as part of nutritional therapy? [Select all that apply]

A. Plan enteral feeding to meet at least 50 percent of calorie requirements.
B. Start enteral nutrition within the first 24 hours.
C. Wait until the patient recovers to start with enteral nutrition.
D. Start a slow continuous drip of small amounts of enteral feedings.
E. Start parenteral nutrition if enteral feedings are contraindicated.

A

B. Start enteral nutrition within the first 24 hours.
D. Start a slow continuous drip of small amounts of enteral feedings.
E. Start parenteral nutrition if enteral feedings are contraindicated.

Enteral nutrition should ideally begin within the first 24 hours. It is important because it enhances the perfusion of the (gastrointestinal) GI tract and helps to maintain the integrity of the gastrointestinal mucosa. Enteral feeding should be started with a slow continuous drip of small amounts of enteral feedings. If enteral feedings are contraindicated, parenteral feedings can be started. Enteral feeding should be planned to meet at least 80 percent of the total calorie requirements, but if it is not feasible, parenteral feeding should be started. Nutritional therapy should start as early as possible. The nurse should not wait for the patient to recover before starting nutritional therapy.

How well did you know this?
1
Not at all
2
3
4
5
Perfectly
5
Q

When caring for a critically ill patient who is being mechanically ventilated, the nurse will astutely monitor for which clinical manifestation of multiple organ dysfunction syndrome (MODS)?

A. Increased serum albumin
B. Decreased respiratory compliance
C. Increased gastrointestinal (GI) motility
D. Decreased blood urea nitrogen (BUN)/creatinine ratio

A

B. Decreased respiratory compliance

Clinical manifestations of MODS include symptoms of respiratory distress, signs and symptoms of decreased renal perfusion, decreased serum albumin and prealbumin, decreased GI motility, acute neurologic changes, myocardial dysfunction, disseminated intravascular coagulation (DIC), and changes in glucose metabolism. Serum albumin is not increased, GI motility decreases in MODS, and the BUN/Creatinine ratio likely will increase.

How well did you know this?
1
Not at all
2
3
4
5
Perfectly
6
Q

What occurs when the inflammatory response is activated in a patient with systemic inflammatory response syndrome (SIRS)?

A. Release of mediators
B. Decrease in metabolism
C. Damage of the mesothelium
D. Decrease in vascular permeability

A

A. Release of mediators

A release of mediators occurs when the inflammatory response is activated. Other changes that occur include an increase in metabolism or hypermetabolism, direct damage to the endothelium, and an increase in vascular permeability.

How well did you know this?
1
Not at all
2
3
4
5
Perfectly
7
Q

Which intervention will the nurse include in the plan of care for a patient who has
cardiogenic shock?

A. Avoid elevating head of bed.
B. Check temperature every 2 hours.
C. Monitor breath sounds frequently.
D. Assess skin for flushing and itching.

A

C. Monitor breath sounds frequently.

Since pulmonary congestion and dyspnea are characteristics of cardiogenic shock, the
nurse should assess the breath sounds frequently. The head of the bed is usually elevated
to decrease dyspnea. Elevated temperature and flushing or itching of the skin are not
typical of cardiogenic shock.

How well did you know this?
1
Not at all
2
3
4
5
Perfectly
8
Q

The following therapies are prescribed by the health care provider for a patient who has respiratory distress and syncope after a bee sting. Which will the nurse administer first?

A. normal saline infusion
B. epinephrine (Adrenalin)
C. dexamethasone (Decadron)
D. diphenhydramine (Benadryl)

A

B. epinephrine (Adrenalin)

Epinephrine rapidly causes peripheral vasoconstriction, dilates the bronchi, and blocks the effects of histamine and reverses the vasodilation, bronchoconstriction, and histamine release that cause the symptoms of anaphylaxis. The other interventions also are appropriate but would not be the first ones administered.

How well did you know this?
1
Not at all
2
3
4
5
Perfectly
9
Q

To evaluate the effectiveness of the pantoprazole (Protonix) ordered for a patient with systemic inflammatory response syndrome (SIRS), which assessment will the nurse perform?

A. Auscultate bowel sounds.
B. Palpate for abdominal pain.
C. Ask the patient about nausea.
D. Check stools for occult blood.

A

D. Check stools for occult blood.

Proton pump inhibitors are given to decrease the risk for stress ulcers in critically ill patients. The other assessments also will be done, but these will not help in determining the effectiveness of the pantoprazole administration.

How well did you know this?
1
Not at all
2
3
4
5
Perfectly
10
Q

A patient with septic shock has a BP of 70/46 mm Hg, pulse 136, respirations 32, temperature 104° F, and blood glucose 246 mg/dL. Which intervention ordered by the health care provider should the nurse implement first?

A. Give normal saline IV at 500 mL/hr.
B. Give acetaminophen (Tylenol) 650 mg rectally.
C. Start insulin drip to maintain blood glucose at 110 to 150 mg/dL.
D. Start norepinephrine (Levophed) to keep systolic blood pressure >90 mm Hg.

A

A. Give normal saline IV at 500 mL/hr.

Because of the low systemic vascular resistance (SVR) associated with septic shock, fluid resuscitation is the initial therapy. The other actions also are appropriate, and should be initiated quickly as well.

How well did you know this?
1
Not at all
2
3
4
5
Perfectly
11
Q

The nurse is caring for a patient who has septic shock. Which assessment finding is most important for the nurse to report to the health care provider?

a. Blood pressure (BP) 92/56 mm Hg
b. Skin cool and clammy
c. Oxygen saturation 92%
d. Heart rate 118 beats/minute

A

b. Skin cool and clammy

Because patients in the early stage of septic shock have warm and dry skin, the patient’s cool and clammy skin indicates that shock is progressing. The other information will also be reported, but does not indicate deterioration of the patient’s status.

How well did you know this?
1
Not at all
2
3
4
5
Perfectly
12
Q

The following interventions are ordered by the health care provider for a patient who has respiratory distress and syncope after eating strawberries. Which will the nurse complete first?

a. Start a normal saline infusion.
b. Give epinephrine (Adrenalin).
c. Start continuous ECG monitoring.
d. Give diphenhydramine (Benadryl).

A

b. Give epinephrine (Adrenalin).

Epinephrine rapidly causes peripheral vasoconstriction, dilates the bronchi, and blocks the effects of histamine and reverses the vasodilation, bronchoconstriction, and histamine release that cause the symptoms of anaphylaxis. The other interventions are also appropriate but would not be the first ones completed.

How well did you know this?
1
Not at all
2
3
4
5
Perfectly
13
Q

A patient experiences a myocardial infarction (MI). The nurse closely monitors the patient for complications and recognizes that hypotension is a warning sign of:

A. A secondary MI
B. Pulmonary edema
C. Cardiogenic shock
D. Fatal dysrhythmias

A

C. Cardiogenic shock

One of the initial cardinal signs of cardiogenic shock after a myocardial infarction (MI) is a slow, steady drop in blood pressure. Hypotension after an MI may be an indirect sign of a secondary MI or a fatal dysrhythmia. Depending on the origin of pulmonary edema, patients may experience hypotension or hypertension.

How well did you know this?
1
Not at all
2
3
4
5
Perfectly
14
Q

A patient is brought to the emergency department (ED) after multiple bee stings. On assessment, the nurse finds that the patient has edema on the lips and tongue as well as chest pain, dizziness, wheezing, and stridor. What type of shock should the nurse document this as?

A. Septic shock
B. Neurogenic shock
C. Anaphylactic shock

A

C. Anaphylactic shock

Anaphylactic shock is an acute, life-threatening hypersensitivity reaction to a sensitizing substance that, in this case, is insect venom. The reaction quickly causes massive vasodilation, release of vasoactive mediators, and an increase in capillary permeability. As capillary permeability increases, fluid leaks from the vascular space into the interstitial space. The consequences of these pathophysiologic processes include edema on the lips and tongue, chest pain, wheezing, and stridor. Sepsis is a systemic inflammatory response to a documented or suspected infection. Neurogenic shock is a hemodynamic phenomenon that can occur within 30 minutes of a spinal cord injury at the fifth thoracic (T5) vertebra or above.

How well did you know this?
1
Not at all
2
3
4
5
Perfectly
15
Q

Why is there a loss of lean body mass in patients with systemic inflammatory response syndrome (SIRS) and multiorgan dysfunction syndrome (MODS)?

A. Hypoglycemia occurs.
B. Glucose is converted to glycogen.
C. Fatty acids are mobilized for fuel.
D. Glucose is converted to amino acids.

A

C. Fatty acids are mobilized for fuel.

Both SIRS and MODS trigger a hypermetabolism response leading to mobilization of fatty acids for fuel. Such a catabolic state leads to loss of lean body mass. Because catecholamines and glucocorticoids are released, hyperglycemia occurs, not hypoglycemia. Hyperglycemia also occurs, because glycogen stores are converted into glucose. Once glycogen is depleted, amino acids are converted into glucose and there is a reduction in protein stores.

How well did you know this?
1
Not at all
2
3
4
5
Perfectly
16
Q

A patient is treated in the emergency department (ED) for shock of unknown etiology. The first action by the nurse should be to:

a. administer oxygen.
b. attach a cardiac monitor.
c. obtain the blood pressure.
d. check the level of consciousness.

A

a. administer oxygen.

The initial actions of the nurse are focused on the ABCs—airway, breathing,
circulation—and administration of oxygen should be done first. The other actions should
be accomplished as rapidly as possible after oxygen administration.

How well did you know this?
1
Not at all
2
3
4
5
Perfectly
17
Q

A massive gastrointestinal bleed has resulted in hypovolemic shock in an older patient. What is a priority nursing diagnosis?

A. Acute pain
B. Impaired tissue integrity
C. Decreased cardiac output
D. Ineffective tissue perfusion

A

D. Ineffective tissue perfusion

The many deleterious effects of shock are all related to inadequate perfusion and oxygenation of every body system. Ineffective tissue perfusion supersedes acute pain, impaired tissue integrity, and decreased cardiac output, because circulation is a priority. Acute pain may occur, but is not a priority at this time. Tissue integrity is not impaired.

How well did you know this?
1
Not at all
2
3
4
5
Perfectly
18
Q

A patient’s localized infection has progressed to the point where septic shock now is suspected. What medication is an appropriate treatment modality for this patient?

A. Insulin infusion
B. Intravenous (IV) administration of epinephrine
C. Aggressive IV crystalloid fluid resuscitation
D. Administration of nitrates and β-adrenergic blockers

A

C. Aggressive IV crystalloid fluid resuscitation

Patients in septic shock require large amounts of crystalloid fluid replacement. Epinephrine is indicated in anaphylactic shock, and insulin infusion is not normally necessary in the treatment of septic shock (but can be). Nitrates and β-adrenergic blockers are used most often in the treatment of patients in cardiogenic shock.

How well did you know this?
1
Not at all
2
3
4
5
Perfectly
19
Q

Which type of shock can be treated by minimizing spinal cord trauma with stabilization?

A. Septic shock
B. Neurogenic shock
C. Anaphylactic shock
D. Hypovolemic shock

A

B. Neurogenic shock

Neurogenic shock is caused by severe injury to the spinal cord and results in loss of sympathetic stimulation of blood vessels. Apart from administering vasoconstrictor agents, minimizing the spinal cord trauma with stabilization is a supporting therapy for neurogenic shock. Septic shock occurs in response to a systemic infection. Obtaining the cultures before starting antibiotics is appropriate care for septic shock. A life-threatening allergic reaction to a sensitizing substance causes anaphylactic shock. Avoiding exposure to allergens is supportive therapy for anaphylactic shock. Excessive loss of intravascular fluid causes hypovolemic shock. Besides restoring fluid volume, correcting the cause of fluid loss is supportive therapy.

How well did you know this?
1
Not at all
2
3
4
5
Perfectly
20
Q

The nurse reviews the medical record of a patient with pneumonia and notes that the patient has hypotension, hypothermia, leukocytosis, and hypoxemia. What should the nurse infer from these findings?

A. The patient has septic shock.
B. The patient has neurogenic shock.
C. The patient has cardiogenic shock.
D. The patient has hypovolemic shock.

A

A. The patient has septic shock.

Septic shock is most commonly found in the patient having gram-negative bacterial infections, such as pneumonia. Because it is characterized by hypertension, hypothermia, leukocytosis, and hypoxemia in patients with infections, the nurse concludes that the patient has septic shock. Neurogenic shock is most commonly seen in the patient who has an injury. Cardiogenic shock is caused by cardiovascular disorders, such as a myocardial infarction and cardiomyopathy. Hypovolemic shock is caused by hemorrhage or trauma.

How well did you know this?
1
Not at all
2
3
4
5
Perfectly
21
Q

When examining a patient with cardiogenic shock, which signs of peripheral hypoperfusion does the nurse expect? Select all that apply.

A. Cyanosis
B. Cold skin
C. Weak pulse
D. Bradycardia
E. Hypertension
A

A. Cyanosis
B. Cold skin
C. Weak pulse

Cyanosis, cold skin, and a weak pulse are the signs of peripheral hypoperfusion in cardiogenic shock. Bradycardia and hypertension are not seen in cardiogenic shock; instead, tachycardia and low blood pressure are noted.

How well did you know this?
1
Not at all
2
3
4
5
Perfectly
22
Q

When examining a patient in the progressive stage of shock, which factors related to the gastrointestinal (GI) system should the nurse consider? Select all that apply.

A. Increased motility and peristalsis
B. Increased likelihood of GI ulcers
C. Increased ability to absorb nutrients
D. Increased risk of GI bleeding
E. Increased risk of bacterial migration from the GI tract to the bloodstream.
A

B. Increased likelihood of GI ulcers
D. Increased risk of GI bleeding
E. Increased risk of bacterial migration from the GI tract to the bloodstream.

In the progressive stage of shock, the GI system gets affected by prolonged decreased tissue perfusion. As the blood supply to the GI tract is decreased, the normally protective mucosal barrier becomes ischemic. This ischemia predisposes the patient to ulcers and GI bleeding. It also increases the risk of bacterial migration from the GI tract to the blood. The decreased perfusion to the GI tract also results in a decreased ability to absorb nutrients, decreased motility, and slowed peristalsis.

How well did you know this?
1
Not at all
2
3
4
5
Perfectly
23
Q

A patient presents to the emergency department (ED) in a state of shock. On assessment, the nurse finds that the patient is cyanotic and has crackles on auscultation of the lungs. As which type of shock will the nurse classify this?

A. Neurogenic shock
B. Cardiogenic shock
C. Hypovolemic shock
D. Anaphylactic shock

A

B. Cardiogenic shock

A patient with cardiogenic shock shows peripheral hypoperfusion presenting as cyanosis and has crackles on auscultation of the lungs due to pulmonary congestion. In neurogenic shock, the patient demonstrates symptoms related to the injury such as hypotension and bradycardia. The patient in hypovolemic shock may experience tachycardia as a late sign. In anaphylactic shock, the patient may experience wheezing and stridor.

How well did you know this?
1
Not at all
2
3
4
5
Perfectly
24
Q

If the patient in shock is to receive 1000 mL of normal saline in two hours, at what rate should the infusion pump be set? Fill in the blank. ___mL/hour

A

500

For the 1000 mL of normal saline to be infused in two hours, the infusion pump should be set at 500 mL per hour (1000 mL divided by two hours).

How well did you know this?
1
Not at all
2
3
4
5
Perfectly
25
Q

The nurse is caring for a patient who developed cardiogenic shock. Which medical diagnosis does the nurse suspect?

A. Urosepsis
B. Hemorrhage
C. Myocardial infarction
D. Tension pneumothorax

A

C. Myocardial infarction

Myocardial infarction may produce necrotic areas of cardiac tissue that lead to impaired contractility and decreased cardiac output. This may lead to a cardiogenic shock state. Hemorrhage may lead to a hypovolemic shock state, tension pneumothorax may lead to an obstructive shock state, and urosepsis may lead to a septic shock state.

How well did you know this?
1
Not at all
2
3
4
5
Perfectly
26
Q

A patient admitted to the hospital after a motor vehicle accident (MVA) is in hypovolemic shock. On examination, the nurse finds that the patient is becoming anxious, and the urine output is decreasing. What appropriate action should the nurse perform?

A. Prepare for administering blood products.
B. Begin crystalloid fluid replacement.
C. Start fluids only if deterioration occurs.
D. Wait for the patient to compensate naturally.

A

B. Begin crystalloid fluid replacement.

When the volume of blood loss is less than 30 percent, crystalloid fluid replacements are performed to reverse tissue dysfunction. Blood products are administered when the blood volume loss is more than 30 percent. The nurse should not wait for deterioration to occur to start the fluid replacement therapy. This is because the body can typically compensate naturally for a blood volume loss up to 15 percent.

How well did you know this?
1
Not at all
2
3
4
5
Perfectly
27
Q

Which type of shock is associated with hyperglycemia, presence of pulmonary infiltrates in chest x-ray and increased levels of blood urea nitrogen (BUN)?

A. Septic
B. Cardiogenic
C. Obstructive
D. Hypovolemic

A

B. Cardiogenic

Increased blood levels of glucose, nitrogen, cardiac markers and presence of pulmonary infiltrates are seen in cardiogenic shock. Increased blood levels of lactate, glucose, and positive blood cultures are signs of septic shock. Manifestations of obstructive shock are specific to the area or organ of obstruction. Electrolyte imbalances and decreased hemoglobin and hematocrit are seen in hypovolemic shock.

How well did you know this?
1
Not at all
2
3
4
5
Perfectly
28
Q

The nurse recalls that cardiogenic shock is differentiated from other forms of shock because the patient with cardiogenic shock typically experiences:

A. Hypotension
B. Dysrhythmias
C. Volume excess
D. Volume depletion

A

C. Volume excess

In cardiogenic shock the heart fails as a pump. This usually results in fluid retention and poor perfusion of organs, including the kidneys, which adds to the fluid volume excess. As cardiogenic shock progresses, hypotension will develop, but it is not the cause of cardiogenic shock. Dysrhythmias may or may not be present with cardiogenic shock. Volume depletion is not generally seen with cardiogenic shock. It is, however, seen more with hypovolemic or distributive forms of shock.

How well did you know this?
1
Not at all
2
3
4
5
Perfectly
29
Q

Which type of shock is associated with bradycardia?

A. Septic shock
B. Neurogenic shock
C. Hypovolemic shock
D. Anaphylactic shock

A

B. Neurogenic shock

Neurogenic shock is associated with bradycardia. Myocardial dysfunction and changes in body temperature are signs of septic shock. Reduction in preload, capillary refill and stroke volume are clinical manifestations of hypovolemic shock. Chest pain is seen in anaphylactic shock.

How well did you know this?
1
Not at all
2
3
4
5
Perfectly
30
Q

A nurse is examining a patient with anaphylactic shock due to an insect bite. What types of skin manifestations would the nurse expect to find? Select all that apply.

A. Pallor
B. Pruritus
C. Flushing
D. Urticaria 
E. Cold, clammy skin
A

B. Pruritus
C. Flushing
D. Urticaria

Insect bites may cause allergic reactions and anaphylactic shock. The skin manifestations may include pruritus, flushing, and urticaria caused by massive vasodilation, release of vasoactive mediators, and an increase in capillary permeability. Pallor and cold, clammy skin changes are usually seen in cardiogenic, hypovolemic, and obstructive shock caused by decreased circulatory volume and tissue perfusion.

How well did you know this?
1
Not at all
2
3
4
5
Perfectly
31
Q

What is a manifestation of the irreversible (refractory) stage of shock?

A. Delirium
B. Areflexia
C. Restlessness
D. Alterations in the level of consciousness

A

B. Areflexia

Areflexia or loss of reflexes is a manifestation of the irreversible stage of shock. The progressive state of shock is associated with delirium. Restlessness and altered levels of consciousness indicate that the patient is in the compensatory stage of shock.

How well did you know this?
1
Not at all
2
3
4
5
Perfectly
32
Q

What laboratory finding correlates with a medical diagnosis of cardiogenic shock?

A. Decreased liver enzymes
B. Increased white blood cells
C. Decreased red blood cells, hemoglobin, and hematocrit
D. Increased blood urea nitrogen (BUN) and serum creatinine levels

A

D. Increased blood urea nitrogen (BUN) and serum creatinine levels

The renal hypoperfusion that accompanies cardiogenic shock results in increased BUN and creatinine levels. Impaired perfusion of the liver results in increased liver enzymes, whereas white blood cell levels typically do not increase in cardiogenic shock. Red blood cell indices are typically normal because of relative hypovolemia.

How well did you know this?
1
Not at all
2
3
4
5
Perfectly
33
Q

When examining a patient with septic shock, what symptoms would the nurse expect to find? Select all that apply.

A. Paralytic ileus
B. Gastrointestinal (GI) bleeding
C. Pulsus paradoxus
D. Distended jugular vein
E. Decreased urinary output
A

A. Paralytic ileus
B. Gastrointestinal (GI) bleeding
E. Decreased urinary output

Patients suffering from septic shock may experience decreased tissue perfusion, which may result in a paralytic ileus, GI bleeding, and decreased urinary output. Pulsus paradoxus and jugular vein distension are found in obstructive shock and are mainly the result of compromised hemodynamics.

How well did you know this?
1
Not at all
2
3
4
5
Perfectly
34
Q

The intensive care unit nurse is caring for a patient who is ventilated mechanically. To prevent sepsis in this patient, which nursing intervention does the nurse include in the plan of care?

A. Provide oral care every two to four hours.
B. Turn patient from side to side every eight hours.
C. Position patient in a supine position every two hours.
D. Use clean gloves when suctioning the endotracheal tube.

A

A. Provide oral care every two to four hours.

Providing oral care every two to four hours is correct, because research has found that the oral flora of critically ill patients are predominately gram-negative organisms that can potentially cause ventilator-associated pneumonia. Oral care will help reduce the organisms. Turning the patient from side to side every eight hours is incorrect, because patients need to be turned at least every two hours to prevent accumulation of mucus, which could lead to pneumonia. Positioning the patient in a supine position is incorrect, because patients should have the head of the bed elevated during mechanical ventilation. The nurse should use sterile gloves when conducting endotracheal suctioning of the patient.

How well did you know this?
1
Not at all
2
3
4
5
Perfectly
35
Q

The nurse reviews the plan of care for a patient with multisystem organ dysfunction syndrome. What is the most desirable outcome for the patient?

A. The patient will be free of signs and symptoms of sepsis
B. The patient will maintain a balanced fluid intake and output
C. The patient will experience enhanced overall well-being and mental rest
D. The patient will demonstrate improved perfusion and oxygenation of organs

A

D. The patient will demonstrate improved perfusion and oxygenation of organs

The underlying pathophysiology of multisystem organ dysfunction syndrome (MODS) is a lack of perfusion to organs, resulting in tissue and/or organ hypoxia. Interventions to improve perfusion with fluids or medications improve patient outcomes. The outcomes listed in the other answer options are appropriate and desirable for the patient with MODS, but they are secondary to improved perfusion and oxygenation.

How well did you know this?
1
Not at all
2
3
4
5
Perfectly
36
Q

Which medical emergency is caused by the failure of two or more organ systems?

A. Crush syndrome
B. Toxic shock syndrome
C. Multiple organ dysfunction syndrome (MODS)
D. Systemic inflammatory response syndrome (SIRS)

A

C. Multiple organ dysfunction syndrome (MODS)

Multiple organ dysfunction syndrome is caused by the failure of two or more organ systems in an acutely ill patient. A crushing injury to the skeletal muscle causes crush syndrome, which is characterized by shock and renal failure. Bacterial toxins cause toxic shock syndrome, which is characterized by high fever, hypotension and malaise. Systemic inflammatory response syndrome (SIRS) is a systemic inflammatory response caused by infection, ischemia, infarction and injury.

How well did you know this?
1
Not at all
2
3
4
5
Perfectly
37
Q

Which drug helps manage renal manifestations in a patient with systemic inflammatory response syndrome (SIRS) and multiple organ dysfunction syndrome (MODS)?

A. Sucralfate
B. Furosemide
C. Omeprazole
D. Acetaminophen

A

B. Furosemide

Furosemide is a loop diuretic that helps to manage renal manifestations in a patient with systemic inflammatory response syndrome (SIRS) and multiple organ dysfunction syndrome (MODS). Sucrafate is administered for prophylaxis against stress ulcers, which are gastrointestinal manifestations of SIRS and MODS. Omeprazole is a proton pump inhibitor that has the same action. Acetaminophen is an antipyretic drug given as an acute intervention to manage fevers in patients who are in shock.

How well did you know this?
1
Not at all
2
3
4
5
Perfectly
38
Q

Which sign of neurologic dysfunction is commonly seen in both systemic inflammatory response syndrome (SIRS) and multiple organ dysfunction syndrome (MODS)?

A. Increased heart rate
B. Increased liver enzymes
C. Difficulty breathing
D. Confusion, agitation, and lethargy

A

D. Confusion, agitation, and lethargy

The sign of neurologic dysfunction commonly seen in systemic inflammatory response syndrome (SIRS) and multiple organ dysfunction syndrome (MODS) is change in mental status, which may cause the patient to become confused, agitated, and lethargic. The patient’s heart rate increases due to changes in the cardiovascular system. The patient’s liver enzymes increase due to dysfunction in the hepatic system and finally the dysfunction results in hepatic encephalopathy. The patient’s dyspnea is caused by changes in the respiratory system by inflammatory mediators.

How well did you know this?
1
Not at all
2
3
4
5
Perfectly
39
Q

A patient is diagnosed with multiple organ dysfunction syndrome. While aggressive treatment is continued, the nurse suspects infection. What is the most appropriate action that the nurse should perform?

A. Discontinue the aggressive treatment.
B. Reduce oxygen delivery to the patient.
C. Wait for laboratory reports to confirm the suspicion.
D. Obtain a prescription for broad-spectrum antibiotic therapy.

A

D. Obtain a prescription for broad-spectrum antibiotic therapy.

If an infection is suspected, broad-spectrum antibiotics should be started immediately to limit the infection. Aggressive treatment for infection control should be carried out in parallel. These patients are usually hypoxemic. Therefore, oxygen should be administered strictly as prescribed. Cultures can be sent, and based on the reports, specific antibiotics can be added.

How well did you know this?
1
Not at all
2
3
4
5
Perfectly
40
Q

A patient is showing signs of anaphylactic shock from an insect sting. Which primary health care provider’s prescription does the nurse implement first?

A. Epinephrine 1:1000, 0.5 mg subcutaneous (SQ)
B. Normal saline intravenous (IV) to run at 150 mL/hr
C. Diphenhydramine 50 mg IV
D. Oxygen via nasal cannula at 3 L

A

A. Epinephrine 1:1000, 0.5 mg subcutaneous (SQ)

The patient in anaphylaxis experiences bronchial spasm and constriction. The administration of epinephrine is necessary to reverse this process and facilitate an open airway. Although administering normal saline, diphenhydramine, and oxygen are appropriate, they must be done after an airway has been established.

How well did you know this?
1
Not at all
2
3
4
5
Perfectly
41
Q

Which system of the body is often the first to show signs of dysfunction in systemic inflammatory response syndrome (SIRS) and multiple organ dysfunction syndrome (MODS)?

A. Neurologic system
B. Respiratory system
C. Cardiovascular system
D. Gastrointestinal system

A

B. Respiratory system

Systemic inflammatory response syndrome (SIRS) and multiple organ dysfunction syndrome (MODS) occur due to a systemic inflammatory response. Inflammatory mediators have a direct effect on the pulmonary vasculature. Thus, the respiratory system is often the first system to show signs of dysfunction. Signs of nervous system dysfunction, such as mental changes, can be early signs of SIRS and MODS. However, the nervous system is not the first system to show signs of SIRS and MODS. When the respiratory system is affected, ventilation-perfusion mismatch becomes worse. Tissue oxygen demands increase, leading to cardiovascular changes. Hence, cardiovascular changes occur after changes in the respiratory system. In the early stages of SIRS and MODS, there is shunting away of blood from the gastrointestinal tract, making it vulnerable to ischemic injury. However, such changes show signs of dysfunction later than does the respiratory system.

How well did you know this?
1
Not at all
2
3
4
5
Perfectly
42
Q

How is systemic inflammatory response syndrome (SIRS) different from multiple organ dysfunction syndrome (MODS)?

A. Shock leads to SIRS, and SIRS causes MODS.
B. MODS is reversible, and SIRS has irreversible changes.
C. SIRS is caused by shock, and MODS is caused by perfusion deficits.
D. Homeostasis fails before SIRS, and homeostasis is maintained in MODS.

A

A. Shock leads to SIRS, and SIRS causes MODS.

Any type of shock triggers the systemic inflammatory response. Generalized inflammation in organs remote from the initial shock is systemic inflammatory response syndrome (SIRS). Multiple organ dysfunction syndrome (MODS) results from SIRS. Both SIRS and MODS are reversible in the early stages. SIRS is caused by some kind of injury to the body such as sepsis, ischemia, infarction, and injury; SIRS can lead to MODS if not treated. In SIRS, a type of shock triggers a systemic inflammatory response, after which the body’s homeostasis fails. In MODS, homeostasis fails and medical intervention is needed.

How well did you know this?
1
Not at all
2
3
4
5
Perfectly
43
Q

The nurse assesses a patient with multisystem organ dysfunction syndrome. What assessment finding is most indicative of deterioration?

A. Arterial PO2 of 95%
B. Pulse rate of 108 beats/minute
C. Total urine output of 120 mL over the past eight hours
D. Auscultation of fine bilateral crackles and a moist cough

A

C. Total urine output of 120 mL over the past eight hours

A decrease in urine output to less than 30 mL/hr in an adult is an early indication of hypoperfusion to the kidneys, as well as other vital organs. This may or may not be accompanied by changes in vital signs. An arterial PO2 of 95%, a pulse rate of 108 beats/minute, and auscultation of crackles and a moist cough may also be indications of early deterioration but are not as specific as a low urinary output in identifying deterioration.

How well did you know this?
1
Not at all
2
3
4
5
Perfectly
44
Q

What is the correct ORDER of the pathophysiologic steps involved in systemic inflammatory response syndrome (SIRS)?

  1. Activation of coagulation cascade
  2. Release of mediators
  3. Increase in vascular permeability
  4. Leakage of mediators and proteins into interstitial space
  5. Digestion of foreign debris by white blood cells
A
  1. Release of mediators
  2. Increase in vascular permeability
  3. Leakage of mediators and proteins into interstitial space
  4. Digestion of foreign debris by white blood cells
  5. Activation of coagulation cascade
How well did you know this?
1
Not at all
2
3
4
5
Perfectly
45
Q

Which cardiovascular change is commonly found in patients with systemic inflammatory response syndrome (SIRS)?

A. Decrease in heart rate
B. Decrease in capillary refill
C. Decrease in central venous pressure
D. Decrease in pulmonary artery wedge pressure

A

B. Decrease in capillary refill

Patients with systemic inflammatory response syndrome (SIRS) have decreased capillary refill. Other cardiovascular changes include increases rather than decreases in heart rate, central venous pressure, and pulmonary artery wedge pressure.

How well did you know this?
1
Not at all
2
3
4
5
Perfectly
46
Q

A nurse caring for a patient with multiple organ dysfunction syndrome understands that the patient may be at increased risk of bleeding. What nursing interventions should the nurse perform to manage this patient? Select all that apply.

A. Observe bleeding sites.
B. Decrease fluid intake.
C. Provide enteral feedings.
D. Administer platelets and clotting factors.
E. Minimize traumatic interventions.
A

A. Observe bleeding sites.
D. Administer platelets and clotting factors.
E. Minimize traumatic interventions.

The patient with multiple organ dysfunction syndrome is at a risk of bleeding due to increased bleeding time, thrombocytopenia, and dysfunctional clotting process. The nursing interventions should be aimed at preventing potential bleeding and replacing factors being lost. The patient should be observed for frank or occult bleeding from potential sites. The factors like platelets and clotting factors should be replaced if deficient. Traumatic interventions such as intramuscular injections or multiple venipunctures should be avoided. Decreasing the fluid intake and providing enteral feedings will not help in minimizing hematologic complications.

How well did you know this?
1
Not at all
2
3
4
5
Perfectly
47
Q

What causes gut bacteria to move into circulation in patients with systemic inflammatory response syndrome (SIRS)?

A. Toxic effects of medicines
B. Effect of inflammatory mediators
C. Decreased gastrointestinal motility
D. Decreased perfusion of gut mucosa

A

D. Decreased perfusion of gut mucosa

Decreased perfusion in the gastrointestinal (GI) tract leads to a breakdown of the normally protective mucosal barrier, which causes the bacterial movement from the GI tract into circulation. In order to control these bacteria, antibiotics are administered. Antibiotics are nephrotoxic medicines that can cause acute kidney injury. The breakdown of the mucosal barrier is the direct effect of hypoperfusion rather than the inflammatory mediators. In critical illnesses, GI motility is often decreased causing abdominal distension and paralytic ileus.

How well did you know this?
1
Not at all
2
3
4
5
Perfectly
48
Q

What type of medication does the nurse anticipate being prescribed by the health care provider to manage confusion, disorientation, and delirium in a patient with systemic inflammatory response syndrome (SIRS) and multiple organ dysfunction syndrome (MODS)?

A. Vasopressors
B. Loop diuretics
C. Proton pump inhibitors
D. Calcium channel blockers

A

D. Calcium channel blockers

Impaired perfusion of the brain may cause confusion, disorientation, and delirium in the patient. The health care provider is likely to prescribe calcium channel blockers to a patient exhibiting confusion, disorientation, and delirium to reduce cerebral vasospasm and improve perfusion of the brain. Vasopressors may be prescribed to combat cardiovascular dysfunction. Loop diuretics are prescribed if there is renal dysfunction. Proton pump inhibitors are prescribed to manage gastrointestinal symptoms.

How well did you know this?
1
Not at all
2
3
4
5
Perfectly
49
Q

What therapy is provided to a patient with acute respiratory distress syndrome (ARDS)?

A. Mechanical ventilation
B. Oxygen via a Venturi mask
C. Oxygen via a non-rebreather mask
D. Small volume nebulizer treatments

A

A. Mechanical ventilation

A patient with acute respiratory distress syndrome (ARDS) would be intubated and receive mechanical ventilation. Small volume nebulizer treatments would open airways; however, this intervention will not sufficiently treat ARDS. Oxygen via a Venturi mask or a non-rebreather mask would be insufficient to promote oxygenation and perfusion.

How well did you know this?
1
Not at all
2
3
4
5
Perfectly
50
Q

Which type of shock causes an absence of bowel sounds?

A. Cardiogenic shock
B. Neurogenic shock
C. Hypovolemic shock
D. Anaphylactic shock

A

C. Hypovolemic shock

Absence of bowel sounds is associated with hypovolemic shock. Decreased bowel sounds are seen with cardiogenic shock. Bowel dysfunction is associated with neurogenic shock. Abdominal pain, nausea and vomiting are seen with anaphylactic shock.

How well did you know this?
1
Not at all
2
3
4
5
Perfectly
51
Q

What is the goal in the care of a systemic inflammatory response syndrome (SIRS) patient whose bilirubin level is 3 mg/dL?

A. Patient will be free of stress ulcers.
B. Patient will not feel abdominal distension.
C. Patient will maintain intraabdominal pressures.
D. Patient will maintain adequate tissue perfusion.

A

D. Patient will maintain adequate tissue perfusion.

The nurse will plan to maintain adequate tissue perfusion for a systemic inflammatory response syndrome (SIRS) patient with bilirubin level of 3 mg/dL. An elevation of the bilirubin indicates impaired liver function. Stress ulcer prophylaxis is routine and would have been initiated before SIRS occurred. Abdominal distention and increased intraabdominal pressures are associated with impaired liver function. Monitoring these conditions is essential but is of a lower priority than maintaining tissue perfusion.

How well did you know this?
1
Not at all
2
3
4
5
Perfectly
52
Q

What causes dysrhythmias in patients with systemic inflammatory response syndrome (SIRS) and multiple organ dysfunction syndrome (MODS)?

A. Metabolic acidosis resulting in increase in lactate levels
B. Increased capillary permeability in the cardiovascular system
C. Hypokalemia due to activities of aldosterone and catecholamines
D. Aldosterone-mediated sodium and water reabsorption in kidneys

A

C. Hypokalemia due to activities of aldosterone and catecholamines

In systemic inflammatory response syndrome (SIRS) and multiple organ dysfunction syndrome (MODS), there are hormonal and metabolic changes and fluid shifts, including the release of aldosterone and catecholamines. Aldosterone increases urinary potassium loss and catecholamines cause potassium to move into the cell, resulting in hypokalemia, which causes dysrhythmias. Dysrhythmias are not associated with an increased lactate level caused by metabolic acidosis. Increased capillary permeability causes tachycardia rather than dysrhythmias. The increase in sodium reabsorption raises the serum osmolality and stimulates the release of the antidiuretic hormone (ADH). Increased sodium is not related to the development of dysrhythmias.

How well did you know this?
1
Not at all
2
3
4
5
Perfectly
53
Q

Which laboratory finding in a patient with multiple organ dysfunction syndrome (MODS) suggests prerenal manifestations of renal dysfunction?

A. Urine Na+ is 22 mEq/L.
B. Urine specific gravity is 1.010.
C. Urine osmolality is decreased.
D. Urine specific gravity is increased.

A

D. Urine specific gravity is increased.

An increase in urine specific gravity suggests prerenal manifestations in patients with MODS. This occurs due to renal hypoperfusion. A urine Na+ level above 20 mEq/L is a sign of intrarenal manifestations in MODS. It occurs due to acute tubular necrosis. Other intrarenal manifestations of MODS are urine specific gravity of around 1.010 and a decrease of the urine osmolality.

How well did you know this?
1
Not at all
2
3
4
5
Perfectly
54
Q

What is the clinical manifestation of systemic inflammatory response syndrome (SIRS) and multiple organ dysfunction syndrome (MODS) on the respiratory system?

A. Pulmonary edema
B. Pulmonary fibrosis
C. Pulmonary embolism
D. Pulmonary hypertension

A

D. Pulmonary hypertension

Systemic inflammatory response syndrome (SIRS) and multiple organ dysfunction syndrome (MODS) affect the respiratory system and lead to pulmonary hypertension. Pulmonary edema is caused by fluid accumulation in the air spaces. Pulmonary fibrosis is a respiratory disorder caused by scars in the lung tissues or inhalation of airborne toxins. A pulmonary embolism is caused by blood clots or blockage in the pulmonary arteries.

How well did you know this?
1
Not at all
2
3
4
5
Perfectly
55
Q

The nurse is administering oxygen therapy to a patient in septic shock. What are the possible factors that directly affect oxygen delivery in the patient and should be monitored? Select all that apply.

A. Urine output
B. White blood cells
C. Cardiac output
D. Available hemoglobin
E. Arterial oxygen saturation
A

C. Cardiac output
D. Available hemoglobin
E. Arterial oxygen saturation

Oxygen delivery depends on cardiac output, available hemoglobin, and arterial oxygen saturation (SaO2). The amount of blood that the heart pumps to the body may decide the amount of oxygen delivered to the tissues. Hemoglobin, the protein content in red blood cells, is responsible for carrying oxygen molecules. Low hemoglobin means a low oxygen supply to the tissues. Arterial oxygen saturation indicates the total oxygen carried by the blood in the arteries and implies the level of tissue oxygenation. The urine output and white blood cells are also important parameters to be monitored in a patient who suffers from a shock. However, these do not impact the oxygen delivery directly.

How well did you know this?
1
Not at all
2
3
4
5
Perfectly
56
Q

The health care provider prescribes a dose of dobutamine for a patient in cardiogenic shock due to myocardial infarction. What appropriate actions should the nurse perform for safely administering the medication?
Select all that apply.

A. Monitor heart rate and blood pressure.
B. Stop infusion if tachydysrhythmias develop.
C. Always administer with sodium bicarbonate.
D. Administer through a central line.
E. Use a glass bottle for infusion.

A

A. Monitor heart rate and blood pressure.
B. Stop infusion if tachydysrhythmias develop.
D. Administer through a central line.

Doubutamine is a sympathomimetic medication. When used in therapy with dobutamine, the patient’s heart rate and blood pressure should be continuously monitored, as they may worsen hypotension, requiring the addition of a vasopressor. The infusion should be stopped if tachydysrhythmias develop. The administration through a central line is recommended, because infiltration leads to tissue sloughing. The drug should not be administered with sodium bicarbonate, because it can get deactivated. Because dobutamine is not adsorbed in plastic containers, it is not necessary to administer the drug in glass bottles.

How well did you know this?
1
Not at all
2
3
4
5
Perfectly
57
Q

The nurse is caring for a patient who is experiencing cardiogenic shock as a result of myocardial infarction. Which nursing assessment finding is most concerning?

A. PaO2 60 mm Hg
B. Blood pressure 100/56 mm Hg
C. Urine output 260 mL in eight hours
D. Heart rate 96 beats/minute

A

A. PaO2 60 mm Hg

A PaO2 of 60 is below the normal 80 to 100 mm Hg. The patient experiencing cardiogenic shock will exhibit hypotension and tachycardia, and therefore a blood pressure of 100/56 mm Hg and heart rate of 96 would not apply. A urine output of 260 cc/8 hrs is borderline but not reportable without a continued trending pattern.

How well did you know this?
1
Not at all
2
3
4
5
Perfectly
58
Q

A patient in shock is receiving 0.9 % NaCl (normal saline solution-NSS). Which nursing intervention is appropriate for this patient?

A. Monitor the patient’s vital signs
B. Monitor for the signs of circulatory overload
C. Monitor for signs of hypernatremia in the patient
D. Monitor for allergic reactions and acute renal failure

A

B. Monitor for the signs of circulatory overload

Circulatory overload occurs due to fluid overload. Continuous infusion of 0.9% NaCl increases the fluid volume in the body and may cause circulatory overload. The patient’s vital signs must be checked during transfusion of blood or blood products because they could cause an infection or an allergic reaction. Hypernatremia occurs when the patient is on 1.8%, 3%, and 5% NaCl infusions. Infusion of dextran-40 has a tendency to precipitate allergic reactions and acute renal failure.

How well did you know this?
1
Not at all
2
3
4
5
Perfectly
59
Q

The primary health care provider prescribes antibiotics and vasopressors for a patient. Which type of shock does the nurse expect to be treating?

A. Septic shock
B. Cardiogenic shock
C. Neurogenic shock
D. Anaphylactic shock

A

A. Septic shock

Septic shock occurs in response to infection. Therefore, antibiotics are prescribed for a patient with septic shock. Cardiogenic shock occurs when systolic or diastolic function of the heart is impaired. Sympathomimetic drugs are used for the treatment of cardiogenic shock. Injury to the spinal cord at the fifth thoracic vertebra or above causes neurogenic shock. Vasconstricting medications are prescribed to prevent vasodilation for a patient in septic shock. Anaphylactic shock is a life-threatening allergic reaction to a sensitizing substance. Antihistamines, bronchodilators, and corticosteroids are used in the treatment of anaphylactic shock.

How well did you know this?
1
Not at all
2
3
4
5
Perfectly
60
Q

While planning the management of oxygen delivery in a patient with shock, what appropriate measures should the nurse undertake? Select all that apply.

A. Encourage the patient to move around to increase lung expansion.
B. Space activities that increase oxygen consumption.
C. Monitor continuously by using a central venous catheter.
D. Space activities that decrease oxygen consumption.
E. Administer supplemental oxygen as prescribed.

A

B. Space activities that increase oxygen consumption.
C. Monitor continuously by using a central venous catheter.
E. Administer supplemental oxygen as prescribed.

To optimize oxygen supply and ventilation in a patient suffering from shock, the activities that increase oxygen consumption should be evenly spaced. Mixed venous oxygen saturation should be monitored through a central venous catheter. The patient should not exert energy by excessive moving around because it increases oxygen demand.

How well did you know this?
1
Not at all
2
3
4
5
Perfectly
61
Q

A pt. has a spinal cord injury at T4. VS include a falling BP with bradycardia. The nurse recognizes that the pt. is experiencing

A) a relative hypervolemia
B) an absolute Hypovolemia
C) Neurogenic shock from low blood flow
D) Neurogenic shock from massive vasodilation

A

D) Neurogenic shock from massive vasodilation

How well did you know this?
1
Not at all
2
3
4
5
Perfectly
62
Q

A 78 y.o. man has confusion and a temp of 104 F. He is a diabetic with purulent drainage from his right heel. After an infusion of 3L of normal saline solution, his assessment findings are BP 84/40, HR 110, RR 42/shallow, CO 8L/min and PAWP 4. This patient’s symptoms are most likely indicative of:

a. sepsis
b. septic shock
c. MODS
d. SIRS

A

B) Septic shock (remember - a high CO is indicative of Sepsis, and a low PAWP is indicative of hypovolemia)

How well did you know this?
1
Not at all
2
3
4
5
Perfectly
63
Q

Appropriate treatment modalities for the mgmt. of carcinogenic shock include (select all that apply)

A) dobutamine to increase myocardial contractility
B) Vasopressors to increase systemic vascular resistance
C) circulatory assist devices such as an intraaortic balloon pump
D) corticosteroids to stabilize the cell wall in the infarcted myocardium
E) Trendelenburg positioning to facilitate venous return and increase preload

A

A) dobutamine to increase myocardial contractility

C) circulatory assist devices such as an intraaortic balloon pump

How well did you know this?
1
Not at all
2
3
4
5
Perfectly
64
Q

The most accurate assessment parameters for the nurse to use to determine adequate tissue perfusion in the pt. with MODS are

A) BP, HR, RR
B) LS, BP, temp
C) pulse pressure, LOC, and papillary response
D) LOC, urine output, and skin color and temp

A

D) LOC, urine output, and skin color and temp

How well did you know this?
1
Not at all
2
3
4
5
Perfectly
65
Q

A patient with septic shock has a urine output of 20 mL/hr for the past 3 hours. The pulse rate is 120 and the central venous pressure and pulmonary artery wedge pressure are low. Which of these orders by the health care provider will the nurse question?

a. Give furosemide (Lasix) 40 mg IV.
b. Increase normal saline infusion to 150 mL/hr.
c. Administer hydrocortisone (SoluCortef) 100 mg IV.
d. Prepare to give drotrecogin alpha (Xigris) 24 mcg/kg/hr.

A

a. Give furosemide (Lasix) 40 mg IV.

Furosemide will lower the filling pressures and renal perfusion further for the patient
with septic shock. The other orders are appropriate.

How well did you know this?
1
Not at all
2
3
4
5
Perfectly
66
Q

A patient with shock of unknown etiology whose hemodynamic monitoring indicates BP 92/54, pulse 64, and an elevated pulmonary artery wedge pressure has the following collaborative interventions prescribed. Which intervention will the nurse question?

a. Infuse normal saline at 250 mL/hr.
b. Keep head of bed elevated to 30 degrees.
c. Give nitroprusside (Nipride) unless systolic BP <90 mm Hg.
d. Administer dobutamine (Dobutrex) to keep systolic BP >90 mm Hg.

A

a. Infuse normal saline at 250 mL/hr.

The patient’s elevated pulmonary artery wedge pressure indicates volume excess. A normal saline infusion at 250 mL/hr will exacerbate this. The other actions are
appropriate for the patient.

How well did you know this?
1
Not at all
2
3
4
5
Perfectly
67
Q

A patient with massive trauma and possible spinal cord injury is admitted to the emergency department (ED). Which finding by the nurse will help confirm a diagnosis of neurogenic shock?

a. Cool, clammy skin
b. Inspiratory crackles
c. Apical heart rate 48 beats/min
d. Temperature 101.2° F (38.4° C)

A

c. Apical heart rate 48 beats/min

Neurogenic shock is characterized by hypotension and bradycardia. The other findings would be more consistent with other types of shock.

How well did you know this?
1
Not at all
2
3
4
5
Perfectly
68
Q

After receiving 1000 mL of normal saline, the central venous pressure for a patient who has septic shock is 10 mm Hg, but the blood pressure is still 82/40 mm Hg. The nurse will anticipate the administration of

a. nitroglycerine (Tridil).
b. drotrecogin alpha (Xigris).
c. norepinephrine (Levophed).
d. sodium nitroprusside (Nipride).

A

c. norepinephrine (Levophed).

When fluid resuscitation is unsuccessful, vasopressor drugs are administered to increase
the systemic vascular resistance (SVR) and improve tissue perfusion. Nitroglycerin
would decrease the preload and further drop cardiac output and BP. Drotrecogin alpha
may decrease inappropriate inflammation and help prevent systemic inflammatory
response syndrome, but it will not directly improve blood pressure. Nitroprusside is an
arterial vasodilator and would further decrease SVR.

How well did you know this?
1
Not at all
2
3
4
5
Perfectly
69
Q

To evaluate the effectiveness of the omeprazole (Prilosec) being administered to a patient with systemic inflammatory response syndrome (SIRS), which assessment will the nurse make?

a. Auscultate bowel sounds.
b. Ask the patient about nausea.
c. Monitor stools for occult blood.
d. Check for abdominal distention.

A

c. Monitor stools for occult blood.

Proton pump inhibitors are given to decrease the risk for stress ulcers in critically ill patients. The other assessments also will be done, but these will not help in determining the effectiveness of the omeprazole administration.

How well did you know this?
1
Not at all
2
3
4
5
Perfectly
70
Q

A patient with cardiogenic shock has the following vital signs: BP 86/50, pulse 126, respirations 30. The PAWP is increased and cardiac output is low. The nurse will
anticipate

a. infusion of 5% human albumin.
b. administration of furosemide (Lasix) IV.
c. titration of an epinephrine (Adrenalin) drip.
d. administration of hydrocortisone (SoluCortef).

A

b. administration of furosemide (Lasix) IV.

The PAWP indicates that the patient’s preload is elevated and furosemide is indicated to reduce the preload and improve cardiac output. Epinephrine would further increase heart
rate and myocardial oxygen demand. Normal saline infusion would increase the PAWP
further. Hydrocortisone might be used for septic or anaphylactic shock.

How well did you know this?
1
Not at all
2
3
4
5
Perfectly
71
Q

The emergency department (ED) receives notification that a patient who has just been in an automobile accident is being transported to your facility with anticipated arrival in 1 minute. In preparation for the patient’s arrival, the nurse will obtain

a. 500 mL of 5% albumin.
b. lactated Ringer’s solution.
c. two 14-gauge IV catheters.
d. dopamine (Intropin) infusion.

A

c. two 14-gauge IV catheters.

A patient with multiple trauma may require fluid resuscitation to prevent or treat
hypovolemic shock, so the nurse will anticipate the need for 2 large bore IV lines to
administer normal saline. Lactated Ringer’s solution should be used cautiously and will
not be ordered until the patient has been assessed for possible liver abnormalities.
Although colloids may sometimes be used for volume expansion, crystalloids should be
used as the initial therapy for fluid resuscitation. Vasopressor infusion is not used as the
initial therapy for hypovolemic shock.

How well did you know this?
1
Not at all
2
3
4
5
Perfectly
72
Q

Which of these findings is the best indicator that the fluid resuscitation for a patient with hypovolemic shock has been successful?

a. Hemoglobin is within normal limits.
b. Urine output is 60 mL over the last hour.
c. Pulmonary artery wedge pressure (PAWP) is normal.
d. Mean arterial pressure (MAP) is 65 mm Hg.

A

b. Urine output is 60 mL over the last hour.

Assessment of end organ perfusion, such as an adequate urine output, is the best indicator
that fluid resuscitation has been successful. The hemoglobin level, PAWP, and MAP are
useful in determining the effects of fluid administration, but they are not as useful as data
indicating good organ perfusion.

How well did you know this?
1
Not at all
2
3
4
5
Perfectly
73
Q

Norepinephrine (Levophed) has been prescribed for a patient who was admitted with dehydration and hypotension. Which patient information indicates that the nurse should consult with the health care provider before administration of the norepinephrine?

a. The patient’s central venous pressure is 3 mm Hg.
b. The patient is receiving low dose dopamine (Intropin).
c. The patient is in sinus tachycardia at 100 to 110 beats/min.
d. The patient has had no urine output since being admitted.

A

a. The patient’s central venous pressure is 3 mm Hg.

Adequate fluid administration is essential before administration of vasopressors to
patients with hypovolemic shock. The patient’s low central venous pressure indicates a
need for more volume replacement. The other patient data are not contraindications to
norepinephrine administration.

How well did you know this?
1
Not at all
2
3
4
5
Perfectly
74
Q

When the nurse is assessing a patient who is receiving a nitroprusside (Nipride) infusion to treat cardiogenic shock, which finding indicates that the medication is effective?

a. No heart murmur is audible.
b. Skin is warm, pink, and dry.
c. Troponin level is decreased.
d. Blood pressure is 90/40 mm Hg.

A

b. Skin is warm, pink, and dry.

Warm, pink, and dry skin indicates that perfusion to tissues is improved. Since
nitroprusside is a vasodilator, the blood pressure may be low even if the medication is
effective. Absence of a heart murmur and a decrease in troponin level are not indicators
of improvement in shock.

How well did you know this?
1
Not at all
2
3
4
5
Perfectly
75
Q

Which assessment is most important for the nurse to make in order to evaluate whether treatment of a patient with anaphylactic shock has been effective?

a. Pulse rate
b. Orientation
c. Blood pressure
d. Oxygen saturation

A

d. Oxygen saturation

Because the airway edema that is associated with anaphylaxis can affect airway and
breathing, the oxygen saturation is the most critical assessment. Improvements in the
other assessments also will be expected with effective treatment of anaphylactic shock.

How well did you know this?
1
Not at all
2
3
4
5
Perfectly
76
Q

Which information obtained by the nurse when caring for a patient who has cardiogenic shock indicates that the patient may be developing multiple organ dysfunction syndrome (MODS)?

a. The patient’s serum creatinine level is elevated.
b. The patient complains of intermittent chest pressure.
c. The patient has crackles throughout both lung fields.
d. The patient’s extremities are cool and pulses are weak.

A

a. The patient’s serum creatinine level is elevated.

The elevated serum creatinine level indicates that the patient has renal failure as well as
heart failure. The crackles, chest pressure, and cool extremities are all consistent with the
patient’s diagnosis of cardiogenic shock.

How well did you know this?
1
Not at all
2
3
4
5
Perfectly
77
Q

A patient with septic shock has a BP of 70/46 mm Hg, pulse 136, respirations 32, temperature 104° F, and blood glucose 246 mg/dL. Which of these prescribed
interventions will the nurse implement first?

a. Give normal saline IV at 500 mL/hr.
b. Infuse drotrecogin- (Xigris) 24 mcg/kg.
c. Start insulin drip to maintain blood glucose at 110 to 150 mg/dL.
d. Titrate norepinephrine (Levophed) to keep mean arterial pressure (MAP) at 65 to 70 mm Hg.

A

a. Give normal saline IV at 500 mL/hr.

Because of the low systemic vascular resistance (SVR) associated with septic shock, fluid
resuscitation is the initial therapy. The other actions also are appropriate and should be
initiated quickly as well.

78
Q

When the charge nurse is evaluating the skills of a new RN, which action by the new RN indicates a need for more education in the care of patients with shock?

a. Placing the pulse oximeter on the ear for a patient with septic shock
b. Keeping the head of the bed flat for a patient with hypovolemic shock
c. Decreasing the room temperature to 68° F for a patient with neurogenic shock
d. Increasing the nitroprusside (Nipride) infusion rate for a patient with a high SVR

A

c. Decreasing the room temperature to 68° F for a patient with neurogenic shock

Patients with neurogenic shock may have poikilothermia. The room temperature should
be kept warm to avoid hypothermia. The other actions by the new RN are appropriate.

79
Q

When caring for a patient who has septic shock, which assessment finding is most important for the nurse to report to the health care provider?

a. BP 92/56 mm Hg
b. Skin cool and clammy
c. Apical pulse 118 beats/min
d. Arterial oxygen saturation 91%

A

b. Skin cool and clammy

Since patients in the early stage of septic shock have warm and dry skin, the patient’s
cool and clammy skin indicates that shock is progressing. The other information also will
be reported, but does not indicate deterioration of the patient’s status.

80
Q

During change-of-shift report, the nurse learns that a patient has been admitted with dehydration and hypotension after having vomiting and diarrhea for 3 days. Which finding is most important for the nurse to report to the health care provider?

a. Decreased bowel sounds
b. Apical pulse 110 beats/min
c. Pale, cool, and dry extremities
d. New onset of confusion and agitation

A

d. New onset of confusion and agitation

The changes in mental status are indicative that the patient is in the progressive stage of shock and that rapid intervention is needed to prevent further deterioration. The other information is consistent with compensatory shock.

81
Q

A patient who has been involved in a motor vehicle crash is admitted to the emergency department (ED) with cool, clammy skin; tachycardia; and hypotension. Which of these prescribed interventions should the nurse implement first?

a. Place the patient on continuous cardiac monitor.
b. Draw blood to type and crossmatch for transfusions.
c. Insert two 14-gauge IV catheters in antecubital space.
d. Administer oxygen at 100% per non-rebreather mask

A

d. Administer oxygen at 100% per non-rebreather mask

The first priority in the initial management of shock is maintenance of the airway and ventilation. Cardiac monitoring, insertion of IV catheters, and obtaining blood for transfusions also should be rapidly accomplished, but only after actions to maximize

82
Q

The patient with neurogenic shock is receiving a phenylephrine (Neo-Synephrine) infusion through a left forearm IV. Which assessment information obtained by the nurse indicates a need for immediate action?

a. The patient’s IV infusion site is cool and pale.
b. The patient has warm, dry skin on the extremities.
c. The patient has an apical pulse rate of 58 beats/min.
d. The patient’s urine output has been 28 mL over the last hour.

A

a. The patient’s IV infusion site is cool and pale.

The coldness and pallor at the infusion site suggest extravasation of the phenylephrine.
The nurse should discontinue the IV and, if possible, infuse the medication into a central
line. An apical pulse of 58 is typical for neurogenic shock but does not indicate an
immediate need for nursing intervention. A 28 mL urinary output over 1 hour would
require the nurse to monitor the output over the next hour, but an immediate change in
therapy is not indicated. Warm, dry skin is consistent with early neurogenic shock, but it
does not indicate a need for a change in therapy or immediate action.

83
Q

Which information about a patient who is receiving vasopressin (Pitressin) to treat septic shock is most important for the nurse to communicate to the heath care provider?

a. The patient’s heart rate is 108 beats/min.
b. The patient is complaining of chest pain.
c. The patient’s peripheral pulses are weak.
d. The patient’s urine output is 15 mL/hr.

A

b. The patient is complaining of chest pain.

Because vasopressin is a potent vasoconstrictor, it may decrease coronary artery perfusion. The other information is consistent with the patient’s diagnosis and should be
reported to the health care provider but does not indicate a need for a change in therapy.

84
Q

A patient with neurogenic shock has just arrived in the emergency department after a diving accident. He has a cervical collar in place. Which of the following actions should the nurse take (select all that apply)?

a. Prepare to administer atropine IV.
b. Obtain baseline body temperature.
c. Prepare for intubation and mechanical ventilation.
d. Administer large volumes of lactated Ringer’s solution.
e. Administer high-flow oxygen (100%) by non-rebreather mask.

A

a. Prepare to administer atropine IV.
b. Obtain baseline body temperature.
c. Prepare for intubation and mechanical ventilation.
e. Administer high-flow oxygen (100%) by non-rebreather mask.

All of the actions are appropriate except to give large volumes of lactated Ringer’s
solution. The patient with neurogenic shock usually has a normal blood volume, and it is
important not to volume overload the patient. In addition, lactated Ringer’s solution is
used cautiously in all shock situations because the failing liver cannot convert lactate to
bicarbonate.

85
Q

The health care provider prescribes these actions for a patient who has possible septic shock with a BP of 70/42 mm Hg and oxygen saturation of 90%. In which order will the nurse implement the actions?

a. Obtain blood and urine cultures.
b. Give vancomycin (Vancocin) 1 g IV.
c. Infuse vasopressin (Pitressin) 0.01 units/min.
d. Administer normal saline 1000 mL over 30 minutes.
e. Titrate oxygen administration to keep O2 saturation >95%.

A

e. Titrate oxygen administration to keep O2 saturation >95%.
d. Administer normal saline 1000 mL over 30 minutes.
c. Infuse vasopressin (Pitressin) 0.01 units/min.
a. Obtain blood and urine cultures.
b. Give vancomycin (Vancocin) 1 g IV.

The initial action for this hypotensive and hypoxemic patient should be to improve the
oxygen saturation, followed by infusion of IV fluids and vasopressors to improve
perfusion. Cultures should be obtained before administration of antibiotics.

86
Q

A 19-year-old patient with massive trauma and possible spinal cord injury is admitted to the emergency department (ED). Which assessment finding by the nurse will help confirm a diagnosis of neurogenic shock?

a. Inspiratory crackles.
b. Cool, clammy extremities.
c. Apical heart rate 45 beats/min.
d. Temperature 101.2° F (38.4° C).

A

c. Apical heart rate 45 beats/min.

Neurogenic shock is characterized by hypotension and bradycardia. The other findings would be more consistent with other types of shock

87
Q

An older patient with cardiogenic shock is cool and clammy and hemodynamic monitoring indicates a high systemic vascular resistance (SVR). Which intervention should the nurse anticipate doing next?

a. Increase the rate for the dopamine (Intropin) infusion.
b. Decrease the rate for the nitroglycerin (Tridil) infusion.
c. Increase the rate for the sodium nitroprusside (Nipride) infusion.
d. Decrease the rate for the 5% dextrose in normal saline (D5/.9 NS) infusion.

A

c. Increase the rate for the sodium nitroprusside (Nipride) infusion.

Nitroprusside is an arterial vasodilator and will decrease the SVR and afterload, which will improve cardiac output. Changes in the D5/.9 NS and nitroglycerin infusions will not directly decrease SVR. Increasing the dopamine will tend to increase SVR.

88
Q

A patient with cardiogenic shock has the following vital signs: BP 102/50, pulse 128, respirations 28. The pulmonary artery wedge pressure (PAWP) is increased and cardiac output is low. The nurse will anticipate an order for which medication?

a. 5% human albumin
b. Furosemide (Lasix) IV
c. Epinephrine (Adrenalin) drip
d. Hydrocortisone (Solu-Cortef)

A

b. Furosemide (Lasix) IV

The PAWP indicates that the patient’s preload is elevated, and furosemide is indicated to reduce the preload and improve cardiac output. Epinephrine would further increase heart rate and myocardial oxygen demand. 5% human albumin would also increase the PAWP. Hydrocortisone might be considered for septic or anaphylactic shock.

89
Q

When the nurse educator is evaluating the skills of a new registered nurse (RN) caring for patients experiencing shock, which action by the new RN indicates a need for more education?

a. Placing the pulse oximeter on the ear for a patient with septic shock
b. Keeping the head of the bed flat for a patient with hypovolemic shock
c. Maintaining a cool room temperature for a patient with neurogenic shock
d. Increasing the nitroprusside infusion rate for a patient with a very high SVR

A

c. Maintaining a cool room temperature for a patient with neurogenic shock

Patients with neurogenic shock may have poikilothermia. The room temperature should be kept warm to avoid hypothermia. The other actions by the new RN are appropriate.

90
Q

After change-of-shift report in the progressive care unit, who should the nurse care for first?

a. Patient who had an inferior myocardial infarction 2 days ago and has crackles in the lung bases
b. Patient with suspected urosepsis who has new orders for urine and blood cultures and antibiotics
c. Patient who had a T5 spinal cord injury 1 week ago and currently has a heart rate of 54 beats/minute
d. Patient admitted with anaphylaxis 3 hours ago who now has clear lung sounds and a blood pressure of 108/58 mm Hg

A

b. Patient with suspected urosepsis who has new orders for urine and blood cultures and antibiotics

Antibiotics should be administered within the first hour for patients who have sepsis or suspected sepsis in order to prevent progression to systemic inflammatory response syndrome (SIRS) and septic shock. The data on the other patients indicate that they are more stable. Crackles heard only at the lung bases do not require immediate intervention in a patient who has had a myocardial infarction. Mild bradycardia does not usually require atropine in patients who have a spinal cord injury. The findings for the patient admitted with anaphylaxis indicate resolution of bronchospasm and hypotension.

91
Q

Which preventive actions by the nurse will help limit the development of systemic inflammatory response syndrome (SIRS) in patients admitted to the hospital (select all that apply)?

a. Use aseptic technique when caring for invasive lines or devices.
b. Ambulate postoperative patients as soon as possible after surgery.
c. Remove indwelling urinary catheters as soon as possible after surgery.
d. Advocate for parenteral nutrition for patients who cannot take oral feedings.
e. Administer prescribed antibiotics within 1 hour for patients with possible sepsis.

A

a. Use aseptic technique when caring for invasive lines or devices.
b. Ambulate postoperative patients as soon as possible after surgery.
c. Remove indwelling urinary catheters as soon as possible after surgery.
e. Administer prescribed antibiotics within 1 hour for patients with possible sepsis.

Because sepsis is the most frequent etiology for SIRS, measures to avoid infection such as removing indwelling urinary catheters as soon as possible, use of aseptic technique, and early ambulation should be included in the plan of care. Adequate nutrition is important in preventing SIRS. Enteral, rather than parenteral, nutrition is preferred when patients are unable to take oral feedings because enteral nutrition helps maintain the integrity of the intestine, thus decreasing infection risk. Antibiotics should be administered within 1 hour after being prescribed to decrease the risk of sepsis progressing to SIRS.

92
Q

When assessing a patient who spilled hot oil on the right leg and foot, the nurse notes dry, pale, and hard skin. The patient states that the burn is not painful. What term would the nurse use to document the burn depth?

a. First-degree skin destruction
b. Full-thickness skin destruction
c. Deep partial-thickness skin destruction
d. Superficial partial-thickness skin destruction

A

b. Full-thickness skin destruction

With full-thickness skin destruction, the appearance is pale and dry or leathery, and the area is painless because of the associated nerve destruction. Erythema, swelling, and blisters point to a deep partial-thickness burn. With superficial partial-thickness burns, the area is red, but no blisters are present. First-degree burns exhibit erythema, blanching, and pain.

93
Q

On admission to the burn unit, a patient with an approximate 25% total body surface area (TBSA) burn has the following initial laboratory results: Hct 58%, Hgb 18.2 mg/dL (172 g/L), serum K+ 4.9 mEq/L (4.8 mmol/L), and serum Na+ 135 mEq/L (135 mmol/L). Which of the following prescribed actions should be the nurse’s priority?

a. Monitoring urine output every 4 hours.
b. Continuing to monitor the laboratory results.
c. Increasing the rate of the ordered IV solution.
d. Typing and crossmatching for a blood transfusion.

A

c. Increasing the rate of the ordered IV solution.

The patient’s laboratory results show hemoconcentration, which may lead to a decrease in blood flow to the microcirculation unless fluid intake is increased. Because the hematocrit and hemoglobin are elevated, a transfusion is inappropriate, although transfusions may be needed after the emergent phase once the patient’s fluid balance has been restored. On admission to a burn unit, the urine output would be monitored more often than every 4 hours (likely every1 hour).

94
Q

A patient is admitted to the burn unit with burns to the head, face, and hands. Initially, wheezes are heard, but an hour later, the lung sounds are decreased and no wheezes are audible. What is the best action for the nurse to take?

a. Encourage the patient to cough and auscultate the lungs again.
b. Notify the health care provider and prepare for endotracheal intubation.
c. Document the results and continue to monitor the patient’s respiratory rate.
d. Reposition the patient in high-Fowler’s position and reassess breath sounds.

A

b. Notify the health care provider and prepare for endotracheal intubation.

The patient’s history and clinical manifestations suggest airway edema, and the health care provider should be notified immediately so that intubation can be done rapidly. Placing the patient in a more upright position or having the patient cough will not address the problem of airway edema. Continuing to monitor is inappropriate because immediate action should occur.

95
Q

A patient has just been admitted with a 40% total body surface area (TBSA) burn injury. To maintain adequate nutrition, the nurse should plan to take which action?

a. Administer vitamins and minerals intravenously.
b. Insert a feeding tube and initiate enteral feedings.
c. Infuse total parenteral nutrition via a central catheter.
d. Encourage an oral intake of at least 5000 kcal per day.

A

b. Insert a feeding tube and initiate enteral feedings.

Enteral feedings can usually be started during the emergent phase at low rates and increased over 24 to 48 hours to the goal rate. During the emergent phase, the patient will be unable to eat enough calories to meet nutritional needs and may have a paralytic ileus that prevents adequate nutrient absorption. Vitamins and minerals may be administered during the emergent phase, but these will not assist in meeting the patient’s caloric needs. Parenteral nutrition increases the infection risk, does not help preserve gastrointestinal function, and is not routinely used in burn patients unless the gastrointestinal tract is not available for use.

96
Q

While the patient’s full-thickness burn wounds to the face are exposed, what nursing action prevents cross contamination?

a. Use sterile gloves when removing dressings.
b. Wear gown, cap, mask, and gloves during care.
c. Keep the room temperature at 70° F (20° C) at all times.
d. Give IV antibiotics to prevent bacterial colonization of wounds.

A

b. Wear gown, cap, mask, and gloves during care.

Use of gowns, caps, masks, and gloves during all patient care will decrease the possibility of wound contamination for a patient whose burns are not covered. When removing contaminated dressings and washing the dirty wound, use nonsterile, disposable gloves. The room temperature should be kept at approximately 85° F for patients with open burn wounds to prevent shivering. Systemic antibiotics are not well absorbed into deep burns because of the lack of circulation.

97
Q

A nurse is caring for a patient who has burns of the ears, head, neck, and right arm and hand. The nurse should place the patient in which position?

a. Place the right arm and hand flexed in a position of comfort.
b. Elevate the right arm and hand on pillows and extend the fingers.
c. Assist the patient to a supine position with a small pillow under the head.
d. Position the patient in a side-lying position with rolled towel under the neck.

A

b. Elevate the right arm and hand on pillows and extend the fingers.

The right hand and arm should be elevated to reduce swelling and the fingers extended to avoid flexion contractures (even though this position may not be comfortable for the patient). The patient with burns of the ears should not use a pillow for the head because this will put pressure on the ears, and the pillow may stick to the ears. Patients with neck burns should not use a pillow or rolled towel because the head should be maintained in an extended position in order to avoid contractures.

98
Q

A patient with circumferential burns of both legs develops a decrease in dorsalis pedis pulse strength and numbness in the toes. Which action should the nurse take first?

a. Monitor the pulses every hour.
b. Notify the health care provider.
c. Elevate both legs above heart level with pillows.
d. Encourage the patient to flex and extend the toes.

A

b. Notify the health care provider.

The decrease in pulse and numbness in a patient with circumferential burns indicates decreased circulation to the legs and the need for an escharotomy. Monitoring the pulses is not an adequate response to the decrease in circulation. Elevating the legs or increasing toe movement will not improve the patient’s circulation.

99
Q

Esomeprazole (Nexium) is prescribed for a patient who incurred extensive burn injuries 5 days ago. Which nursing assessment would best evaluate the effectiveness of the drug?

a. Bowel sounds
b. Stool frequency
c. Stool occult blood
d. Abdominal distention

A

c. Stool occult blood

H2 blockers and proton pump inhibitors are given to prevent Curling’s ulcer in the patient who has sustained burn injuries. Proton pump inhibitors usually do not affect bowel sounds, stool frequency, or appetite.

100
Q

Which prescribed drug is best for the nurse to give before scheduled wound debridement on a patient with partial-thickness burns?

a. ketorolac
b. lorazepam (Ativan)
c. gabapentin (Neurontin)
d. hydromorphone (Dilaudid)

A

d. hydromorphone (Dilaudid)

Opioid pain medications are the best choice for pain control. The other drugs are used as adjuvants to enhance the effects of opioids.

101
Q

A young adult patient who is in the rehabilitation phase after having deep partial-thickness face and neck burns has a nursing diagnosis of disturbed body image. Which statement by the patient best indicates that the problem is resolving?

a. “I’m glad the scars are only temporary.”
b. “I will avoid using a pillow, so my neck will be OK.”
c. “Do you think dark beige makeup will cover this scar?”
d. “I don’t think my boyfriend will want to look at me now.”

A

c. “Do you think dark beige makeup will cover this scar?”

The willingness to use strategies to enhance appearance is an indication that the disturbed body image is resolving. Expressing feelings about the scars indicates a willingness to discuss appearance but not resolution of the problem. Because deep partial-thickness burns leave permanent scars, a statement that the scars are temporary indicates denial rather than resolution of the problem. Avoiding using a pillow will help prevent contractures, but it does not address the problem of disturbed body image.

102
Q

The nurse caring for a patient admitted with burns over 30% of the body surface assesses that urine output has dramatically increased. Which action by the nurse would best support maintaining kidney function?

a. Monitor white blood cells (WBCs).
b. Continue to measure the urine output.
c. Assess that blisters and edema have subsided.
d. Encourage the patient to eat an adequate number of calories.

A

b. Continue to measure the urine output.

The patient’s urine output indicates that the patient is entering the acute phase of the burn injury and moving on from the emergent stage. At the end of the emergent phase, capillary permeability normalizes, and the patient begins to diurese large amounts of urine with a low specific gravity. Although this may occur at about 48 hours, it may be longer in some patients. Blisters and edema begin to resolve, but this process requires more time. WBCs may increase or decrease, based on the patient’s immune status and any infectious processes. The WBC count does not indicate kidney function. Although adequate nutrition is important for healing, it does not ensure adequate kidney functioning.

103
Q

A patient with burns covering 40% total body surface area (TBSA) is in the acute phase of burn treatment. Which snack would be best for the nurse to offer to this patient?

a. Bananas
b. Orange gelatin
c. Vanilla milkshake
d. Whole grain bagel

A

c. Vanilla milkshake

A patient with a burn injury needs high-protein and high-calorie food intake, and the milkshake is the highest in these nutrients. The other choices are not as nutrient dense as the milkshake. Gelatin is likely high in sugar. The bagel is a good carbohydrate choice but low in protein. Bananas are a good source of potassium but are not high in protein and calories

104
Q

A patient has just arrived in the emergency department after an electrical burn from exposure to a high-voltage current. What is the priority nursing assessment?

a. Oral temperature
b. Peripheral pulses
c. Extremity movement
d. Pupil reaction to light

A

c. Extremity movement

With chemical burns, the initial action is to remove the chemical from contact with the skin as quickly as possible. Remove nonadherent clothing, shoes, watches, jewelry, glasses, or contact lenses (if the face was exposed). Flush the chemical from the wound and surrounding area with copious amounts of saline solution or water. Covering the affected area or placing cool compresses on the area will leave the chemical in contact with the skin. Application of an alkaline solution is not recommended.

105
Q

A patient who has burns on the arms, legs, and chest from a house fire has become agitated and restless 8 hours after being admitted to the hospital. Which action should the nurse take first?

a. Stay at the bedside and reassure the patient.
b. Administer the ordered morphine sulfate IV.
c. Assess orientation and level of consciousness.
d. Use pulse oximetry to check oxygen saturation.

A

d. Use pulse oximetry to check oxygen saturation.

Agitation in a patient who may have suffered inhalation injury might indicate hypoxia, and this should be assessed by the nurse first. Administration of morphine may be indicated if the nurse determines that the agitation is caused by pain. Assessing level of consciousness and orientation is also appropriate but not as essential as determining whether the patient is hypoxemic. Reassurance is not helpful to reduce agitation in a hypoxemic patient.

106
Q

A patient arrives in the emergency department with facial and chest burns caused by a house fire. Which action should the nurse take first?

a. Auscultate the patient’s lung sounds.
b. Determine the extent and depth of the burns.
c. Give the prescribed hydromorphone (Dilaudid).
d. Infuse the prescribed lactated Ringer’s solution

A

a. Auscultate the patient’s lung sounds.

A patient with facial and chest burns is at risk for inhalation injury and assessment of airway and breathing is the priority. The other actions will be completed after airway management is assured.

107
Q

A patient with extensive electrical burn injuries is admitted to the emergency department. Which prescribed intervention should the nurse implement first?

a. Assess pain level.
b. Place on heart monitor.
c. Check potassium level.
d. Assess oral temperature.

A

b. Place on heart monitor.

After an electrical burn, the patient is at risk for life-threatening dysrhythmias and should be placed on a heart monitor. Assessing the oral temperature and pain is not as important as assessing for cardiac dysrhythmias. Checking the potassium level is important, but it will take time before the laboratory results are back. The first intervention is to place the patient on a heart monitor and assess for dysrhythmias so that they can be monitored and treated if necessary.

108
Q

Eight hours after a thermal burn covering 50% of a patient’s total body surface area (TBSA), the nurse assesses the patient. The patient weighs 92 kg (202.4 lb). Which information would be a priority to communicate to the health care provider?

a. Blood pressure is 95/48 per arterial line.
b. Urine output of 41 mL over past 2 hours.
c. Serous exudate is leaking from the burns.
d. Heart monitor shows sinus tachycardia of 108.

A

b. Urine output of 41 mL over past 2 hours.

The urine output should be at least 0.5 to 1.0 mL/kg/hr during the emergent phase, when the patient is at great risk for hypovolemic shock. The nurse should notify the health care provider because a higher IV fluid rate is needed. BP during the emergent phase should be greater than 90 mm Hg systolic and the pulse rate should be less than 120 beats/min. Serous exudate from the burns is expected during the emergent phase.

109
Q

Which patient should the nurse assess first?

a. A patient with burns who is complaining of level 8 (0 to 10 scale) pain
b. A patient with smoke inhalation who has wheezes and altered mental status
c. A patient with full-thickness leg burns who is scheduled for a dressing change
d. A patient with partial thickness burns who is receiving IV fluids at 500 mL/hr

A

b. A patient with smoke inhalation who has wheezes and altered mental status

This patient has evidence of lower airway injury and hypoxemia, and should be assessed immediately to determine the need for O2 or intubation (or both). The other patients should also be assessed as rapidly as possible, but they do not have evidence of life-threatening complications.

110
Q

Which patient is most appropriate for the burn unit charge nurse to assign to a registered nurse (RN) who has floated from the hospital medical unit?

a. A patient who has twice-daily burn debridements to partial-thickness facial burns
b. A patient who has just returned from having a cultured epithelial autograft to the chest
c. A patient who has a weight loss of 15% from admission and will have enteral feedings started
d. A patient who has blebs under an autograft on the thigh and has an order for bleb aspiration

A

c. A patient who has a weight loss of 15% from admission and will have enteral feedings started

An RN from a medical unit would be familiar with malnutrition and with administration and evaluation of response to enteral feedings. The other patients require burn assessment and care that is more appropriate for staff who regularly care for burned patients.

111
Q

A patient who was found unconscious in a burning house is brought to the emergency department by ambulance. The nurse notes that the patient’s skin color is bright red. Which action should the nurse take first?

a. Insert two large-bore IV lines.
b. Check the patient’s orientation.
c. Assess for singed nasal hair and dark oral mucous membranes.
d. Place the patient on 100% O2using a nonrebreather mask

A

d. Place the patient on 100% O2using a nonrebreather mask

The patient’s history and skin color suggest carbon monoxide poisoning, which should be treated by rapidly starting O2 at 100%. The other actions can be taken after the action to correct gas exchange.

112
Q

The nurse is reviewing laboratory results on a patient who had a large burn 48 hours ago. Which result requires priority action by the nurse?

a. Hematocrit of 53%
b. Serum sodium of 147 mEq/L
c. Serum potassium of 6.1 mEq/L
d. Blood urea nitrogen of 37 mg/dL

A

c. Serum potassium of 6.1 mEq/L

Hyperkalemia can lead to life-threatening dysrhythmias and indicates that the patient requires cardiac monitoring and immediate treatment to lower the potassium level. The other laboratory values are also abnormal and require changes in treatment, but they are not as immediately life threatening as the elevated potassium level.

113
Q

The charge nurse observes the following actions being taken by a new nurse on the burn unit. Which action by the new nurse would require immediate intervention by the charge nurse?

a. The new nurse uses clean gloves when applying antibacterial cream to a burn wound.
b. The new nurse obtains burn cultures when the patient has a temperature of 95.2° F (35.1° C).
c. The new nurse gives PRN fentanyl (Sublimaze) IV to a patient 5 minutes before a dressing change.
d. The new nurse calls the health care provider when a nondiabetic patient’s serum glucose is elevated.

A

a. The new nurse uses clean gloves when applying antibacterial

Sterile gloves should be worn when applying medications or dressings to a burn. Hypothermia is an indicator of possible sepsis, and cultures are appropriate. Nondiabetic patients may require insulin because stress and high calorie intake may lead to temporary hyperglycemia. Fentanyl peaks 5 minutes after IV administration and should be used just before and during dressing changes for pain management.

114
Q

Which nursing action is a priority for a patient who has suffered a burn injury while working on an electrical power line?

a. Inspect the contact burns.
b. Check the blood pressure.
c. Stabilize the cervical spine.
d. Assess alertness and orientation.

A

c. Stabilize the cervical spine.

Cervical spine injuries are commonly associated with electrical burns. Therefore stabilization of the cervical spine takes precedence after airway management. The other actions are also included in the emergent care after electrical burns, but the most important action is to avoid spinal cord injury.

115
Q

Which action will the nurse include in the plan of care for a patient in the rehabilitation phase after a burn injury to the right arm and chest?

a. Keep the right arm in a position of comfort.
b. Avoid the use of sustained-release narcotics.
c. Teach about the purpose of tetanus immunization.
d. Apply water-based cream to burned areas frequently.

A

d. Apply water-based cream to burned areas frequently.

Application of water-based emollients will moisturize new skin and decrease flakiness and itching. To avoid contractures, the joints of the right arm should be positioned in an extended position, which is not the position of comfort. Patients may need to continue the use of opioids during rehabilitation. Tetanus immunization would have been given during the emergent phase of the burn injury.

116
Q

A young adult patient who is in the rehabilitation phase 6 months after a severe face and neck burn tells the nurse, “I’m sorry that I’m still alive. My life will never be normal again.” Which response by the nurse is best?

a. “Most people recover after a burn and feel satisfied with their lives.”
b. “It’s true that your life may be different. What concerns you the most?”
c. “Why do you feel that way? It will get better as your recovery progresses.”
d. “It is really too early to know how much your life will be changed by the burn.”

A

b. “It’s true that your life may be different. What concerns you the most?”

This response acknowledges the patient’s feelings and asks for more assessment data that will help in developing an appropriate plan of care to assist the patient with the emotional response to the burn injury. The other statements are accurate but do not acknowledge the anxiety and depression that the patient is expressing.

117
Q

In which order will the nurse take these actions when doing a dressing change for a partial-thickness burn wound on a patient’s chest?

a. Apply sterile gauze dressing.
b. Document wound appearance.
c. Apply silver sulfadiazine cream.
d. Give IV fentanyl (Sublimaze).
e. Clean wound with saline-soaked gauze.

A

d. Give IV fentanyl (Sublimaze).
e. Clean wound with saline-soaked gauze.
c. Apply silver sulfadiazine cream.
a. Apply sterile gauze dressing.
b. Document wound appearance.

Because partial-thickness burns are very painful, the nurse’s first action should be to give pain medications. The wound will then be cleaned, antibacterial cream applied, and covered with a new sterile dressing. The last action should be to document the appearance of the wound.

118
Q

A nurse in the ICU is planning the care of a patient who is being treated for shock. Which of the following statements best describes the pathophysiology of this patients health problem?

A) Blood is shunted from vital organs to peripheral areas of the body.
B) Cells lack an adequate blood supply and are deprived of oxygen and nutrients.
C) Circulating blood volume is decreased with a resulting change in the osmotic pressure gradient.
D) Hemorrhage occurs as a result of trauma, depriving vital organs of adequate perfusion.

A

B) Cells lack an adequate blood supply and are deprived of oxygen and nutrients.

Shock is a life-threatening condition with a variety of underlying causes. Shock is caused when the cells have a lack of adequate blood supply and are deprived of oxygen and nutrients. In cases of shock, blood is shunted from peripheral areas of the body to the vital organs. Hemorrhage and decreased blood volume are associated with some, but not all, types of shock.

119
Q

The emergency nurse is admitting a patient experiencing a GI bleed who is believed to be in the compensatory stage of shock. What assessment finding would be most consistent with the early stage of compensation?

A) Increased urine output
B) Decreased heart rate
C) Hyperactive bowel sounds
D) Cool, clammy skin

A

D) Cool, clammy skin

In the compensatory stage of shock, the body shunts blood from the organs, such as the skin and kidneys, to the brain and heart to ensure adequate blood supply. As a result, the patients skin is cool and clammy. Also in this compensatory stage, blood vessels vasoconstrict, the heart rate increases, bowel sounds are hypoactive, and the urine output decreases.

120
Q

The nurse is transferring a patient who is in the progressive stage of shock into ICU from the medical unit. The medical nurse is aware that shock affects many organ systems and that nursing management of the patient will focus on what intervention?

A) Reviewing the cause of shock and prioritizing the patients psychosocial needs
B) Assessing and understanding shock and the significant changes in assessment data to guide the plan of care
C) Giving the prescribed treatment, but shifting focus to providing family time as the patient is unlikely to survive
D) Promoting the patients coping skills in an effort to better deal with the physiologic changes accompanying shock

A

B) Assessing and understanding shock and the significant changes in assessment data to guide the plan of care

Nursing care of patients in the progressive stage of shock requires expertise in assessing and understanding shock and the significance of changes in assessment data. Early interventions are essential to the survival of patients in shock; thus, suspecting that a patient may be in shock and reporting subtle changes in assessment are imperative. Psychosocial needs, such as coping, are important considerations, but they are not prioritized over physiologic health.

121
Q

The nurse in the ICU is admitting a 57-year-old man with a diagnosis of possible septic shock. The nurses assessment reveals that the patient has a normal blood pressure, increased heart rate, decreased bowel sounds, and cold, clammy skin. The nurses analysis of these data should lead to what preliminary conclusion?

A) The patient is in the compensatory stage of shock.
B) The patient is in the progressive stage of shock.
C) The patient will stabilize and be released by tomorrow.
D) The patient is in the irreversible stage of shock.

A

A) The patient is in the compensatory stage of shock.

In the compensatory stage of shock, the blood pressure remains within normal limits. Vasoconstriction, increased heart rate, and increased contractility of the heart contribute to maintaining adequate cardiac output. Patients display the often-described fight or flight response. The body shunts blood from organs such as the skin, kidneys, and GI tract to the brain and heart to ensure adequate blood supply to these vital organs. As a result, the skin is cool and clammy, and bowel sounds are hypoactive. In progressive shock, the blood pressure drops. In septic shock, the patients chance of survival is low and he will certainly not be released within 24 hours. If the patient were in the irreversible stage of shock, his blood pressure would be very low and his organs would be failing.

122
Q

A critical care nurse is aware of similarities and differences between the treatments for different types of shock. Which of the following interventions is used in all types of shock?

A) Aggressive hypoglycemic control
B) Administration of hypertonic IV fluids
C) Early provision of nutritional support
D) Aggressive antibiotic therapy

A

C) Early provision of nutritional support.

Nutritional support is necessary for all patients who are experiencing shock. Hyperglycemic (not hypoglycemic) control is needed for many patients. Hypertonic IV fluids are not normally utilized and antibiotics are necessary only in patients with septic shock.

123
Q

When caring for a patient in shock, one of the major nursing goals is to reduce the risk that the patient will develop complications of shock. How can the nurse best achieve this goal?

A) Provide a detailed diagnosis and plan of care in order to promote the patients and family’s coping
B) Keep the physician updated with the most accurate information because in cases of shock the nurse often cannot provide relevant interventions.
C) Monitor for significant changes and evaluate patient outcomes on a scheduled basis focusing on blood pressure and skin temperature.
D) Understand the underlying mechanisms of shock, recognize the subtle and more obvious signs, and then provide rapid assessment.

A

D) Understand the underlying mechanisms of shock, recognize the subtle and more obvious signs, and then provide rapid assessment.

Shock is a life-threatening condition with a variety of underlying causes. It is critical that the nurse apply the nursing process as the guide for care. Shock is unpredictable and rapidly changing so the nurse must understand the underlying mechanisms of shock. The nurse must also be able to recognize the subtle as well as more obvious signs and then provide rapid assessment and response to provide the patient with the best chance for recovery. Coping skills are important, but not the ultimate priority. Keeping the physician updated with the most accurate information is important, but the nurse is in the best position to provide rapid assessment and response, which gives the patient the best chance for survival. Monitoring for significant changes is critical, and evaluating patient outcomes is always a part of the nursing process, but the subtle signs and symptoms of shock are as important as the more obvious signs, such as blood pressure and skin temperature. Assessment must lead to diagnosis and interventions.

124
Q

In the compensatory stage of hypovolemic shock, to what organs does blood flow decrease after the sympathetic nervous system activates α-adrenergic stimulation? (Select all that apply)

a. skin
b. brain
c. heart
d. kidneys
e. gastrointestinal tract

A

a. skin
d. kidneys
e. gastrointestinal tract

After sympathetic nervous system activation of vasoconstriction, blood flow to nonvitalorgans, such as skin, kidneys, and the gastrointestinal (GI) tract is diverted or shunted to the mostessential organs of the heart and the brain. The patient will feel cool and clammy, the renin-angiotensin-aldosterone system will be activated, and the patient may develop a paralytic ileus

125
Q

What is a serious condition that occurs when an overwhelming bacterial infection affects the body?

A

septic shock

126
Q

The nurse is caring for a 29-yr-old man who was admitted 1 week ago with multiple rib fractures, pulmonary contusions, and a left femur fracture from a motor vehicle crash. The attending physician states the patient has developed sepsis, and the family members have many questions. Which information should the nurse include when explaining the early stage of sepsis?

a. Antibiotics are not useful when an infection has progressed to sepsis.
b. Weaning the patient away from the ventilator is the top priority in sepsis.
c. Large amounts of IV fluid are required in sepsis to fill dilated blood vessels.
d. The patient has recovered from sepsis if he has warm skin and ruddy cheeks.

A

c. Large amounts of IV fluid are required in sepsis to fill dilated blood vessels.

Patients with sepsis may be normovolemic, but because of acute vasodilation, relative hypovolemia and hypotension occur. Patients in septic shock require large amounts of fluid replacement and may require frequent fluid boluses to maintain circulation. Antibiotics are an important component of therapy for patients with septic shock. They should be started after cultures (e.g., blood, urine) are obtained and within the first hour of septic shock. Oxygenating the tissues is the top priority in sepsis, so efforts to wean septic patients from mechanical ventilation halt until sepsis is resolving. Additional respiratory support may be needed during sepsis. Although cool and clammy skin is present in other early shock states, the patient in early septic shock may feel warm and flushed because of a hyperdynamic state.

127
Q

An immunocompromised older adult has developed a urinary tract infection and the care team recognizes the need to prevent an exacerbation of the patients infection that could result in urosepsis and septic shock. What action should the nurse perform to reduce the patients risk of septic shock?

A) Apply an antibiotic ointment to the patients mucous membranes, as ordered
B) Perform passive range-of-motion exercises unless contraindicated
C) Initiate total parenteral nutrition (TPN)
D) Remove invasive devices as soon as they are no longer needed

A

D) Remove invasive devices as soon as they are no longer needed

Early removal of invasive devices can reduce the incidence of infections. Broad application of antibiotic ointments is not performed. TPN may be needed, but this does not directly reduce the risk of further infection. Range-of-motion exercises are not a relevant intervention.

128
Q

The nurse would recognize which clinical manifestation as suggestive of sepsis?

a. Sudden diuresis unrelated to drug therapy
b. Hyperglycemia in the absence of diabetes
c. Respiratory rate of seven breaths per minute
d. Bradycardia with sudden increase in blood pressure

A

b. Hyperglycemia in the absence of diabetes

Hyperglycemia in patients with no history of diabetes is a diagnostic criterion for sepsis. Oliguria, not diuresis, typically accompanies sepsis along with tachypnea and tachycardia.

129
Q

A patient’s localized infection has become systemic and septic shock is suspected. What medication is expected to treat septic shock refractory to fluids?

a. Insulin infusion
b. Furosemide (Lasix) IV push
c. Norepinephrine administered by titration
d. Administration of nitrates and β-adrenergic blockers

A

c. Norepinephrine administered by titration

If fluid resuscitation using crystalloids is not effective, vasopressor medications such as norepinephrine (Levophed) and dopamine are indicated to restore mean arterial pressure (MAP). Nitrates and β-adrenergic blockers are most often used in the treatment of patients in cardiogenic shock. Furosemide (Lasix) is indicated for patients with fluid volume overload. Insulin infusion may be administered to normalize blood sugar and improve overall outcomes, but it is not considered a medication used to treat shock.

130
Q

The nurse is caring for a patient whose progressing infection places her at high risk for shock. What assessment finding would the nurse consider a potential sign of shock?

A) Elevated systolic blood pressure
B) Elevated mean arterial pressure (MAP)
C) Shallow, rapid respirations
D) Bradycardia

A

C) Shallow, rapid respirations

A symptom of shock is shallow, rapid respirations. Systolic blood pressure drops in shock, and MAP is less than 65 mm Hg. Bradycardia occurs in neurogenic shock; other states of shock have tachycardia as a symptom. Infection can lead to septic shock.

131
Q

Which preventive actions by the nurse will help limit the development of systemic inflammatory response syndrome (SIRS) in patients admitted to the hospital (select all that apply)?

a. Ambulate postoperative patients as soon as possible after surgery.
b. Use aseptic technique when manipulating invasive lines or devices.
c. Remove indwelling urinary catheters as soon as possible after surgery.
d. Administer prescribed antibiotics within 1 hour for patients with possible sepsis.
e. Advocate for parenteral nutrition for patients who cannot take in adequate calories.

A

a. Ambulate postoperative patients as soon as possible after surgery.
b. Use aseptic technique when manipulating invasive lines or devices.
c. Remove indwelling urinary catheters as soon as possible after surgery.
d. Administer prescribed antibiotics within 1 hour for patients with possible sepsis.

Because sepsis is the most frequent etiology for SIRS, measures to avoid infection such as removing indwelling urinary catheters as soon as possible, use of aseptic technique, and early ambulation should be included in the plan of care. Adequate nutrition is important in preventing SIRS. Enteral, rather than parenteral, nutrition is preferred when patients are unable to take oral feedings because enteral nutrition helps maintain the integrity of the intestine, thus decreasing infection risk. Antibiotics should be given within 1 hour after being prescribed to decrease the risk of sepsis progressing to SIRS.

132
Q

A 78-kg patient with septic shock has a pulse rate of 120 beats/min with low central venous pressure and pulmonary artery wedge pressure. Urine output has been 30 mL/hr for the past 3 hours. Which order by the health care provider should the nurse question?

a. Administer furosemide (Lasix) 40 mg IV.
b. Increase normal saline infusion to 250 mL/hr.
c. Give hydrocortisone (Solu-Cortef) 100 mg IV.
d. Titrate norepinephrine to keep systolic blood pressure (BP) above 90 mm Hg.

A

a. Administer furosemide (Lasix) 40 mg IV.

Furosemide will lower the filling pressures and renal perfusion further for the patient with septic shock. Patients in septic shock require large amounts of fluid replacement. If the patient remains hypotensive after initial volume resuscitation with minimally 30 mL/kg, vasopressors such as norepinephrine may be added. IV corticosteroids may be considered for patients in septic shock who cannot maintain an adequate BP with vasopressor therapy despite fluid resuscitation.

133
Q

After receiving 2 L of normal saline, the central venous pressure for a patient who has septic shock is 10 mm Hg, but the blood pressure is still 82/40 mm Hg. The nurse will anticipate an order for

a. furosemide.
b. nitroglycerin.
c. norepinephrine
d. sodium nitroprusside.

A

c. norepinephrine

When fluid resuscitation is unsuccessful, vasopressor drugs are given to increase the systemic vascular resistance (SVR) and blood pressure and improve tissue perfusion. Furosemide would cause diuresis and further decrease the BP. Nitroglycerin would decrease the preload and further drop cardiac output and BP. Nitroprusside is an arterial vasodilator and would further decrease SVR.

134
Q

The nurse suspects sepsis as a cause of shock when the laboratory test results indicate

a. hypokalemia
b. thrombocytopenia
c. decreased hemoglobin
d. increased blood urea nitrogen (BUN)

A

b. Thrombocytopenia can occur.

When sepsis is the cause of shock, endotoxin stimulates a cascade of inflammatory responses that start with the release of tumor necrosis factor (TNF) and interleukin-1 (IL-1), which stimulate other inflammatory mediators. The release of platelet- activating factor causes formation of microthrombi and vessel obstruction. There is vasodilation, increased capillary permeability, and neutrophil and platelet aggregation and adhesion to the endothelium. The process does not occur in other types of shock until late stages of shock.

135
Q

A 70-year-old patient is malnourished, has a history of type 2 diabetes mellitus, and is admitted from the nursing home with pneumonia. For which kind of shock should the nurse closely monitor this patient?

a. Septic shock
b. Neurogenic shock
c. Cardiogenic shock
d. Anaphylactic shock

A

a. Septic shock

Older adults with chronic diseases and malnourished or debilitated patients are at risk of developing septic shock, especially when they have an infection (e.g., pneumonia, urinary tract infection) or indwelling lines or catheters.

136
Q

In an acute care setting, the nurse is assessing an unstable patient. When prioritizing the patients care, the nurse should recognize that the patient is at risk for hypovolemic shock in which of the following circumstances?

A) Fluid volume circulating in the blood vessels decreases.
B) There is an uncontrolled increase in cardiac output.
C) Blood pressure regulation becomes irregular.
D) The patient experiences tachycardia and a bounding pulse.

A

A) Fluid volume circulating in the blood vessels decreases.

Hypovolemic shock is characterized by a decrease in intravascular volume. Cardiac output is decreased, blood pressure decreases, and pulse is fast, but weak.

137
Q

A 64-year-old woman is admitted to the emergency department vomiting bright red blood. The patient’s vital signs are blood pressure 78/58 mm Hg, pulse 124 beats/minute, respirations 28 breaths/minute, and temperature 97.2° F (36.2° C). Which physician order should the nurse complete first?

a-Obtain a 12-lead ECG and arterial blood gases
b-Rapidly administer 1000 mL normal saline solution IV.
c-Administer norepinephrine (Levophed) by continuous IV infusion
.d-Carefully insert a nasogastric tube and an indwelling bladder catheter.

A

b-Rapidly administer 1000 mL normal saline solution IV.

Isotonic crystalloids, such as normal saline solution, should be used in the initial resuscitation of hypovolemic shock. Vasopressor drugs (e.g., norepinephrine) may be considered if the patient does not respond to fluid resuscitation and blood products. Other orders (e.g., insertion of nasogastric tube and indwelling bladder catheter and obtaining the diagnostic studies) can be initiated after fluid resuscitation is initiated

138
Q

The nurse is caring for a patient who is exhibiting signs and symptoms of hypovolemic shock following injuries suffered in a motor vehicle accident. The nurse anticipates that the physician will promptly order the administration of a crystalloid IV solution to restore intravascular volume. In addition to normal saline, which crystalloid fluid is commonly used to treat hypovolemic shock?

A) Lactated Ringers
B) Albumin
C) Dextran
D) 3% NaCl

A

A) Lactated Ringers

Crystalloids are electrolyte solutions used for the treatment of hypovolemic shock. Lactated Ringers and 0.9% sodium chloride are isotonic crystalloid fluids commonly used to manage hypovolemic shock. Dextran and albumin are colloids, but Dextran, even as a colloid, is not indicated for the treatment of hypovolemic shock. 3% NaCl is a hypertonic solution and is not isotonic.

139
Q

A massive gastrointestinal bleed has resulted in hypovolemic shock in an older patient. What is a priority nursing diagnosis?

a-Acute pain
b-Impaired tissue integrity
c-Decreased cardiac output
d-Ineffective tissue perfusion

A

d-Ineffective tissue perfusion

The many deleterious effects of shock are all related to inadequate perfusion and oxygenation of every body system. This nursing diagnosis supersedes the other diagnoses.

140
Q

Which finding is the best indicator that the fluid resuscitation for a 90-kg patient with hypovolemic shock has been effective?

a. Hemoglobin is within normal limits.
b. Urine output is 65 mL over the past hour
c. Central venous pressure (CVP) is normal.
d. Mean arterial pressure (MAP) is 72 mm Hg.

A

b. Urine output is 65 mL over the past hour

Assessment of end organ perfusion, such as an adequate urine output, is the best indicator that fluid resuscitation has been successful. Urine output should be equal to or more than 0.5 mL/kg/hr. The hemoglobin level, CVP, and MAP are useful in determining the effects of fluid administration, but they are not as useful as data indicating good organ perfusion.

141
Q

Norepinephrine has been prescribed for a patient who was admitted with dehydration and hypotension. Which patient data indicate that the nurse should consult with the health care provider before starting the norepinephrine?

a. The patient is receiving low dose dopamine.
b. The patient’s central venous pressure is 3 mm Hg.
c. The patient is in sinus tachycardia at 120 beats/min.
d. The patient has had no urine output since being admitted.

A

b. The patient’s central venous pressure is 3 mm Hg.

Adequate fluid administration is essential before giving vasopressors to patients with hypovolemic shock. The patient’s low central venous pressure indicates a need for more volume replacement. The other patient data are not contraindications to norepinephrine administration.

142
Q

Which data collected by the nurse caring for a patient who has cardiogenic shock indicate that the patient may be developing multiple organ dysfunction syndrome (MODS)?

a. The patient’s serum creatinine level is elevated.
b. The patient complains of intermittent chest pressure.
c. The patient’s extremities are cool and pulses are weak.
d. The patient has bilateral crackles throughout lung fields.

A

a. The patient’s serum creatinine level is elevated.

The elevated serum creatinine level indicates that the patient has renal failure as well as heart failure. The crackles, chest pressure, and cool extremities are all symptoms consistent with the patient’s diagnosis of cardiogenic shock.

143
Q

A nurse is caring for a patient whose hemodynamic monitoring indicates a blood pressure of 92/54 mm Hg, a pulse of 64 beats/min, and an elevated pulmonary artery wedge pressure (PAWP). Which intervention ordered by the health care provider should the nurse question?

a. Elevate head of bed to 30 degrees.
b. Infuse normal saline at 250 mL/hr.
c. Hold nitroprusside if systolic BP is less than 90 mm Hg
d. Titrate dobutamine to keep systolic BP is greater than 90 mm Hg..

A

b. Infuse normal saline at 250 mL/hr.

The patient’s elevated PAWP indicates volume excess in relation to cardiac pumping ability, consistent with cardiogenic shock. A saline infusion at 250 mL/hr will exacerbate the volume excess. The other actions will help to improve cardiac output, which should lower the PAWP and may raise the BP.

144
Q

An older patient with cardiogenic shock is cool and clammy. Hemodynamic monitoring indicates a high systemic vascular resistance (SVR). Which intervention should the nurse anticipate?

a. Increase the rate for the dopamine infusion.
b. Decrease the rate for the nitroglycerin infusion.
c. Increase the rate for the sodium nitroprusside infusion.
d. Decrease the rate for the 5% dextrose in normal saline (D5/.9 NS) infusion.

A

c. Increase the rate for the sodium nitroprusside infusion.

Nitroprusside is an arterial vasodilator and will decrease the SVR and afterload, which will improve cardiac output. Changes in the D5/.9 NS and nitroglycerin infusions will not directly decrease SVR. Increasing the dopamine will tend to increase SVR.

145
Q

A patient with cardiogenic shock has the following vital signs: BP 102/50, pulse 128, respirations 28. The pulmonary artery wedge pressure (PAWP) is increased, and cardiac output is low. The nurse will anticipate an order for which medication?

a. 5% albumin infusion
b. furosemide (Lasix) IV
c. epinephrine (Adrenalin) drip
d. hydrocortisone (Solu-Cortef

A

b. furosemide (Lasix) IV

The PAWP indicates that the patient’s preload is elevated, and furosemide is indicated to reduce the preload and improve cardiac output. Epinephrine would further increase the heart rate and myocardial oxygen demand. 5% albumin would also increase the PAWP. Hydrocortisone might be considered for septic or anaphylactic shock.

146
Q

The nurse is caring for a 72-year-old man in cardiogenic shock after an acute myocardial infarction. Which clinical manifestations would be of most concern to the nurse?

a-Restlessness, heart rate of 124 beats/minute, and hypoactive bowel sounds
b-Mean arterial pressure of 54 mm Hg, increased jaundice, and cold, clammy skin
c-PaO2 of 38 mm Hg, serum lactate level of 46.5 mcg/dL, and bleeding from puncture sites
d-Agitation, respiratory rate of 32 breaths/minute, and serum creatinine level of 2.6 mg/dL

A

C- Severe hypoxemia, lactic acidosis, and bleeding are clinical manifestations of the irreversible state of shock. Recovery from this stage is not likely because of multiple organ system failure. Restlessness, tachycardia, and hypoactive bowel sounds are clinical manifestations that occur during the compensatory stage of shock. Decreased mean arterial pressure, jaundice, cold/ clammy skin, agitation, tachypnea, and increased serum creatinine are clinical manifestations of the progressive stage of shock.

147
Q

The nurse is caring for a patient admitted with cardiogenic shock. The patient is experiencing chest pain and there is an order for the administration of morphine. In addition to pain control, what is the main rationale for administering morphine to this patient?

A) It promotes coping and slows catecholamine release.
B) It stimulates the patient so he or she is more alert.
C) It decreases gastric secretions.
D) It dilates the blood vessels.

A

D) It dilates the blood vessels.

For patients experiencing chest pain, morphine is the drug of choice because it dilates the blood vessels and controls the patients anxiety. Morphine would not be ordered to promote coping or to stimulate the patient. The rationale behind using morphine would not be to decrease gastric secretions.

148
Q

A nurse in the ICU receives report from the nurse in the ED about a new patient being admitted with a neck injury he received while diving into a lake. The ED nurse reports that his blood pressure is 85/54, heart rate is 53 beats per minute, and his skin is warm and dry. What does the ICU nurse recognize that that patient is probably experiencing?

A) Anaphylactic shock
B) Neurogenic shock
C) Septic shock
D) Hypovolemic shock

A

B) Neurogenic shock

Neurogenic shock can be caused by spinal cord injury. The patient will present with a low blood pressure; bradycardia; and warm, dry skin due to the loss of sympathetic muscle tone and increased parasympathetic stimulation. Anaphylactic shock is caused by an identifiable offending agent, such as a bee sting. Septic shock is caused by bacteremia in the blood and presents with a tachycardia. Hypovolemic shock presents with tachycardia and a probable source of blood loss.

149
Q

A team of nurses are reviewing the similarities and differences between the different classifications of shock. Which subclassifications of circulatory shock should the nurses identify? Select all that apply

A) Anaphylactic
B) Hypovolemic
C) Cardiogenic
D) Septic
E) Neurogenic
A

A) Anaphylactic
D) Septic
E) Neurogenic

The varied mechanisms leading to the initial vasodilation in circulatory shock provide the basis for the further subclassification of shock into three types: septic shock, neurogenic shock, and anaphylactic shock. Hypovolemic and cardiogenic shock are not subclassifications of circulatory shock.

150
Q

The ICU nurse is caring for a patient in neurogenic shock following an overdose of antianxiety medication. When assessing this patient, the nurse should recognize what characteristic of neurogenic shock?

A) Hypertension
B) Cool, moist skin
C) Bradycardia
D) Signs of sympathetic stimulation

A

C) Bradycardia

In neurogenic shock, the sympathetic system is not able to respond to body stressors. Therefore, the clinical characteristics of neurogenic shock are signs of parasympathetic stimulation. It is characterized by dry, warm skin rather than the cool, moist skin seen in hypovolemic shock. Another characteristic is hypotension with bradycardia, rather than the tachycardia that characterizes other forms of shock.

151
Q

What is a severe shock caused by an allergic reaction?

A

anaphylactic shock

152
Q

A 50-yr-old woman with a suspected brain tumor is scheduled for a CT scan with contrast media. The nurse notifies the physician that the patient reported an allergy to shellfish. Which response by the physician should the nurse question?

a. Infuse IV diphenhydramine before the procedure.
b. Administer lorazepam (Ativan) before the procedure
c. Complete the CT scan without the use of contrast media
d. Premedicate with hydrocortisone sodium succinate (Solu-Cortef).

A

b. Administer lorazepam (Ativan) before the procedure

Administer lorazepam (Ativan) before the procedure. An individual with an allergy to shellfish is at an increased risk to develop anaphylactic shock if contrast media is injected for a CT scan. To prevent anaphylactic shock, the nurse should always confirm the patient’s allergies before diagnostic procedures (e.g., CT scan with contrast media). Appropriate interventions may include cancelling the procedure, completing the procedure without contrast media, or premedication with diphenhydramine or hydrocortisone. IV fluids may be given to promote renal clearance of the contrast media and prevent renal toxicity and acute kidney injury. The use of an antianxiety agent such as lorazepam would not be effective in preventing an allergic reaction to the contrast media.

153
Q

Which interventions should be used for anaphylactic shock (select all that apply)?

a. Antibiotics
b. Vasodilator
c. Antihistamine
d. Oxygen supplementation
e. Colloid volume expansion
f. Crystalloid volume expansion

A

c. Antihistamine
d. Oxygen supplementation
e. Colloid volume expansion

Due to the massive vasodilation, release of vasoactive mediators, and increased in capillary permeability from the immediate reaction, fluid leaks from the vascular space into the interstitial space. By administering a colloid (which contain larger particles that do not penetrate the semipermeable membrane), the large particles will stay intravascularly. Due to their smaller size particle composition, a crystalloid would not stay intravascularly and leak interstitially.

154
Q

The most accurate assessment parameters for the nurse to use to determine adequate tissue perfusion in the patient with MODS are

a. blood pressure, pulse, and respirations
b. breath sounds, blood pressure, and body temperature.
c. pulse pressure, level of consciousness, and pupillary response.
d. level of consciousness, urine output, and skin color and temperature.

A

d. level of consciousness, urine output, and skin color and temperature.

Rationale: Adequate tissue perfusion in a patient with multiple-organ dysfunction syndrome is assessed by the level of consciousness, urine output, capillary refill, peripheral sensation, skin color, extremity skin temperature, and peripheral pulses.

155
Q

The nurse is assisting in the care of several patients in the critical care unit. Which patient is at greatest risk for developing multiple organ dysfunction syndrome (MODS)?

a-22-year-old patient with systemic lupus erythematosus who is admitted with a pelvic fracture after a motor vehicle accident
b-48-year-old patient with lung cancer who is admitted for syndrome of inappropriate antidiuretic hormone and hyponatremia
c-65-year-old patient with coronary artery disease, dyslipidemia, and primary hypertension who is admitted for unstable angina
d-82-year-old patient with type 2 diabetes mellitus and chronic kidney disease who is admitted for peritonitis related to a peritoneal dialysis catheter infection

A

d-82-year-old patient with type 2 diabetes mellitus and chronic kidney disease who is admitted for peritonitis related to a peritoneal dialysis catheter infection

A patient with peritonitis is at high risk for developing sepsis. In addition, a patient with diabetes is at high risk for infections and impaired healing. Sepsis and septic shock are the most common causes of MODS. Individuals at greatest risk for developing MODS are older adults and persons with significant tissue injury or preexisting disease. MODS can be initiated by any severe injury or disease process that activates a massive systemic inflammatory response.

156
Q

The nurse is caring for a patient in the ICU who has been diagnosed with multiple organ dysfunction syndrome (MODS). The nurses plan of care should include which of the following interventions?

A) Encouraging the family to stay hopeful and educating them to the fact that, in nearly all cases, the prognosis is good
B) Encouraging the family to leave the hospital and to take time for themselves as acute care of MODS patients may last for several months
C) Promoting communication with the patient and family along with addressing end-of-life issues
D) Discussing organ donation on a number of different occasions to allow the family time to adjust to the idea

A

C) Promoting communication with the patient and family along with addressing end-of-life issues

Promoting communication with the patient and family is a critical role of the nurse with a patient in progressive shock. It is also important that the health care team address end-of-life decisions to ensure that supportive therapies are congruent with the patients wishes. Many cases of MODS result in death and the life expectancy of patients with MODS is usually measured in hours and possibly days, but not in months. Organ donation should be offered as an option on one occasion, and then allow the family time to discuss and return to the health care providers with an answer following the death of the patient.

157
Q

A patient who is in shock is receiving dopamine in addition to IV fluids. What principle should inform the nurses care planning during the administration of a vasoactive drug?

A) The drug should be discontinued immediately after blood pressure increases.
B) The drug dose should be tapered down once vital signs improve.
C) The patient should have arterial blood gases drawn every 10 minutes during treatment.
D) The infusion rate should be titrated according the patients subjective sensation of adequate perfusion.

A

B) The drug dose should be tapered down once vital signs improve

When vasoactive medications are discontinued, they should never be stopped abruptly because this could cause severe hemodynamic instability, perpetuating the shock state. Subjective assessment data are secondary to objective data. Arterial blood gases should be carefully monitored, but every10-minute draws are not the norm.

158
Q

The intensive care nurse caring for a patient in shock is planning assessments and interventions related to the patients nutritional needs. What physiologic process contributes to these increased nutritional needs?

A) The use of albumin as an energy source by the body because of the need for increased adenosine triphosphate (ATP)
B) The loss of fluids due to decreased skin integrity and decreased stomach acids due to increased parasympathetic activity
C) The release of catecholamines that creates an increase in metabolic rate and caloric requirements
D) The increase in GI peristalsis during shock and the resulting diarrhea

A

C) The release of catecholamines that creates an increase in metabolic rate and caloric requirements

Nutritional support is an important aspect of care for patients in shock. Patients in shock may require 3,000 calories daily. This caloric need is directly related to the release of catecholamines and the resulting increase in metabolic rate and caloric requirements. Albumin is not primarily metabolized as an energy source. The special nutritional needs of shock are not related to increased parasympathetic activity, but are instead related to increased sympathetic activity. GI function does not increase during shock.

159
Q

The nurse is transferring a patient who is in the progressive stage of shock into ICU from the medical unit. The medical nurse is aware that shock affects many organ systems and that nursing management of the patient will focus on what intervention?

A) Reviewing the cause of shock and prioritizing the patients psychosocial needs
B) Assessing and understanding shock and the significant changes in assessment data to guide the plan of care
C) Giving the prescribed treatment, but shifting focus to providing family time as the patient is unlikely to survive
D) Promoting the patients coping skills in an effort to better deal with the physiologic changes accompanying shock

A

B) Assessing and understanding shock and the significant changes in assessment data to guide the plan of care

Nursing care of patients in the progressive stage of shock requires expertise in assessing and understanding shock and the significance of changes in assessment data. Early interventions are essential to the survival of patients in shock; thus, suspecting that a patient may be in shock and reporting subtle changes in assessment are imperative. Psychosocial needs, such as coping, are important considerations, but they are not prioritized over physiologic health.

160
Q

The acute care nurse is providing care for an adult patient who is in hypovolemic shock. The nurse recognizes that antidiuretic hormone (ADH) plays a significant role in this health problem. What assessment finding will the nurse likely observe related to the role of the ADH during hypovolemic shock?

A) Increased hunger
B) Decreased thirst
C) Decreased urinary output
D) Increased capillary perfusion

A

C) Decreased urinary output

During hypovolemic shock, a state of hypernatremia occurs. Hypernatremia stimulates the release of ADH by the pituitary gland. ADH causes the kidneys to retain water further in an effort to raise blood volume and blood pressure. In a hypovolemic state the body shifts blood away from anything that is not a vital organ, so hunger is not an issue; thirst is increased as the body tries to increase fluid volume; and capillary profusion decreases as the body shunts blood away from the periphery and to the vital organs.

161
Q

You are precepting a new graduate nurse in the ICU. You are collaborating in the care of a patient who is receiving large volumes of crystalloid fluid to treat hypovolemic shock. In light of this intervention, for what sign would you teach the new nurse to monitor the patient?

A) Hypothermia
B) Bradycardia
C) Coffee ground emesis
D) Pain

A

A) Hypothermia

Temperature should be monitored closely to ensure that rapid fluid resuscitation does not precipitate hypothermia. IV fluids may need to be warmed during the administration of large volumes. The nurse should monitor the patient for cardiovascular overload and pulmonary edema when large volumes of IV solution are administered. Coffee ground emesis is an indication of a GI bleed, not shock. Pain is related to cardiogenic shock.

162
Q

The nurse is caring for a patient in the ICU whose condition is deteriorating. The nurse receives orders to initiate an infusion of dopamine. What would be the priority assessment and interventions specific to the administration of vasoactive medications?

A) Frequent monitoring of vital signs, monitoring the central line site, and providing accurate drug titration
B) Reviewing medications, performing a focused cardiovascular assessment, and providing patient education
C) Reviewing the laboratory findings, monitoring urine output, and assessing for peripheral edema
D) Routine monitoring of vital signs, monitoring the peripheral IV site, and providing early discharge instructions

A

A) Frequent monitoring of vital signs, monitoring the central line site, and providing accurate drug titration

When vasoactive medications are administered, vital signs must be monitored frequently (at least every 15 minutes until stable, or more often if indicated). Vasoactive medications should be administered through a central venous line because infiltration and extravasation of some vasoactive medications can cause tissue necrosis and sloughing. An IV pump should be used to ensure that the medications are delivered safely and accurately. Individual medication dosages are usually titrated by the nurse, who adjusts drip rates based on the prescribed dose and the patients response. Reviewing medications, performing a focused cardiovascular assessment, and providing patient education are important nursing tasks, but they are not specific to the administration of IV vasoactive drugs. Reviewing the laboratory findings, monitoring urine output, and assessing for peripheral edema are not the priorities for administration of IV vasoactive drugs. Vital signs are taken on a frequent basis when monitoring administration of IV vasoactive drugs, vasoactive medications should be administered through a central venous line, and early discharge instructions would be inappropriate in this time of crisis.

163
Q

The nurse, a member of the health care team in the ED, is caring for a patient who is determined to be in the irreversible stage of shock. What would be the most appropriate nursing intervention?

A) Provide opportunities for the family to spend time with the patient, and help them to understand the irreversible stage of shock.
B) Inform the patients family immediately that the patient will likely not survive to allow the family time to make plans and move forward.
C) Closely monitor fluid replacement therapy, and inform the family that the patient will probably survive and return to normal life.
D) Protect the patients airway, optimize intravascular volume, and initiate the early rehabilitation process.

A

A) Provide opportunities for the family to spend time with the patient, and help them to understand the irreversible stage of shock.

The irreversible (or refractory) stage of shock represents the point along the shock continuum at which organ damage is so severe that the patient does not respond to treatment and cannot survive. Providing opportunities for the family to spend time with the patient and helping them to understand the irreversible stage of shock is the best intervention. Informing the patients family early that the patient will likely not survive does allow the family to make plans and move forward, but informing the family too early will rob the family of hope and interrupt the grieving process. The chance of surviving the irreversible (or refractory) stage of shock is very small, and the nurse needs to help the family cope with the reality of the situation. With the chances of survival so small, the priorities shift from aggressive treatment and safety to addressing the end-of-life issues.

164
Q

The nurse in the ICU is caring for a 47-year-old, obese male patient who is in shock following a motor vehicle accident. The nurse is aware that patients in shock possess excess energy requirements. What would be the main challenge in meeting this patients elevated energy requirements during prolonged rehabilitation?

A) Loss of adipose tissue
B) Loss of skeletal muscle
C) Inability to convert adipose tissue to energy
D) Inability to maintain normal body mass

A

B) Loss of skeletal muscle

Nutritional energy requirements are met by breaking down lean body mass. In this catabolic process, skeletal muscle mass is broken down even when the patient has large stores of fat or adipose tissue. Loss of skeletal muscle greatly prolongs the patients recovery time. Loss of adipose tissue, the inability to convert adipose tissue to energy, and the inability to maintain normal body mass are not main concerns in meeting nutritional energy requirements for this patient.

165
Q

The nurse in the ED is caring for a patient recently admitted with a likely myocardial infarction. The nurse understands that the patients heart is pumping an inadequate supply of oxygen to the tissues. For what health problem should the nurse assess?

A) Dysrhythmias
B) Increase in blood pressure
C) Increase in heart rate
D) Decrease in oxygen demands

A

A) Dysrhythmias

Cardiogenic shock occurs when the hearts ability to pump blood is impaired and the supply of oxygen is inadequate for the heart and tissues. Symptoms of cardiogenic shock include angina pain and dysrhythmias. Cardiogenic shock does not cause increased blood pressure, increased heart rate, or a decrease in oxygen demands.

166
Q

The nurse is providing care for a patient who is in shock after massive blood loss from a workplace injury. The nurse recognizes that many of the findings from the most recent assessment are due to compensatory mechanisms. What is a compensatory mechanism to increase cardiac output during hypovolemic states?

A) Third spacing of fluid
B) Dysrhythmias
C) Tachycardia
D) Gastric hypermotility

A

C) Tachycardia

Tachycardia is a primary compensatory mechanism to increase cardiac output during hypovolemic states. The third spacing of fluid takes fluid out of the vascular space. Gastric hypermotility and dysrhythmias would not increase cardiac output and are not considered to be compensatory mechanisms.

167
Q

The intensive care nurse is responsible for the care of a patient with shock. What cardiac signs or symptoms would suggest to the nurse that the patient may be experiencing acute organ dysfunction? Select all that apply.

A) Drop in systolic blood pressure of 40 mm Hg from baselines
B) Hypotension that responds to bolus fluid resuscitation
C) Exaggerated response to vasoactive medications
D) Serum lactate >4 mmol/L
E) Mean arterial pressure (MAP) of 65 mm Hg

A

A) Drop in systolic blood pressure of 40 mm Hg from baselines
D) Serum lactate >4 mmol/L
E) Mean arterial pressure (MAP) of 65 mm Hg

Signs of acute organ dysfunction in the cardiovascular system include systolic blood pressure <90 mmHg or mean arterial pressure (MAP) <65 mm Hg, drop in systolic blood pressure >40 mm Hg from baselines or serum lactate >4 mmol/L. An exaggerated response to vasoactive medications and an adequate response to fluid resuscitation would not be noted.

168
Q

An adult patient has survived an episode of shock and will be discharged home to finish the recovery phase of his disease process. The home health nurse plays an integral part in monitoring this patient. What aspect of his care should be prioritized by the home health nurse?

A) Providing supervision to home health aides in providing necessary patient care
B) Assisting the patient and family to identify and mobilize community resources
C) Providing ongoing medical care during the family’s rehabilitation phase
D) Reinforcing the importance of continuous assessment with the family

A

B) Assisting the patient and family to identify and mobilize community resources

The home care nurse reinforces the importance of continuing medical care and helps the patient and family identify and mobilize community resources. The home health nurse is part of a team that provides patient care in the home. The nurse does not directly supervise home health aides. The nurse provides nursing care to both the patient and family, not just the family. The nurse performs continuous and ongoing assessment of the patient; he or she does not just reinforce the importance of that assessment.

169
Q

In all types of shock, nutritional demands increase rapidly as the body depletes its stores of glycogen. Enteral nutrition is the preferred method of meeting these increasing energy demands. What is the basis for enteral nutrition being the preferred method of meeting the body’s needs?

A) It slows the proliferation of bacteria and viruses during shock.
B) It decreases the energy expended through the functioning of the GI system.
C) It assists in expanding the intravascular volume of the body.
D) It promotes GI function through direct exposure to nutrients.

A

D) It promotes GI function through direct exposure to nutrients.

Parenteral or enteral nutritional support should be initiated as soon as possible. Enteral nutrition is preferred, promoting GI function through direct exposure to nutrients and limiting infectious complications associated with parenteral feeding. Enteral feeding does not decrease the proliferation of microorganisms or the amount of energy expended through the functioning of the GI system and it does not assist in expanding the intravascular volume of the body.

170
Q

The ICU nurse is caring for a patient with multiple organ dysfunction syndrome (MODS) due to shock. What nursing action should be prioritized at this point during care?

A) Providing information and support to family members
B) Preparing the family for a long recovery process
C) Educating the patient regarding the use of supportive fluids
D) Facilitating the rehabilitation phase of treatment

A

A) Providing information and support to family members

Providing information and support to family members is a critical role of the nurse. Most patients with MODS do not recover, so the rehabilitation phase of recovery is not a short-term priority. Educating the patient about the use of supportive fluids is not a high priority.

171
Q

A critical care nurse is planning assessments in the knowledge that patients in shock are vulnerable to developing fluid replacement complications. For what signs and symptoms should the nurse monitor the patient? Select all that apply.

A) Hypovolemia
B) Difficulty breathing
C) Cardiovascular overload
D) Pulmonary edema
E) Hypoglycemia
A

B) Difficulty breathing
C) Cardiovascular overload
D) Pulmonary edema

Fluid replacement complications can occur, often when large volumes are administered rapidly. Therefore, the nurse monitors the patient closely for cardiovascular overload, signs of difficulty breathing, and pulmonary edema. Hypovolemia is what necessitates fluid replacement, and hypoglycemia is not a central concern with fluid replacement.

172
Q

When circulatory shock occurs, there is massive vasodilation causing pooling of the blood in the periphery of the body. An ICU nurse caring for a patient in circulatory shock should know that the pooling of blood in the periphery leads to what pathophysiological effect?

A) Increased stroke volume
B) Increased cardiac output
C) Decreased heart rate
D) Decreased venous return

A

D) Decreased venous return

Pooling of blood in the periphery results in decreased venous return. Decreased venous return results in decreased stroke volume and decreased cardiac output. Decreased cardiac output, in turn, causes decreased blood pressure and, ultimately, decreased tissue perfusion. Heart rate increases in an attempt to meet the demands of the body.

173
Q

A triage nurse in the ED is on shift when a grandfather carries his 4-year-old grandson into the ED. The child is not breathing, and the grandfather states the boy was stung by a bee in a nearby park while they were waiting for the boys mother to get off work. Which of the following would lead the nurse to suspect that the boy is experiencing anaphylactic shock?

A) Rapid onset of acute hypertension
B) Rapid onset of respiratory distress
C) Rapid onset of neurologic compensation
D) Rapid onset of cardiac arrest

A

B) Rapid onset of respiratory distress

Characteristics of severe anaphylaxis usually include rapid onset of hypotension, neurologic compromise, and respiratory distress. Cardiac arrest can occur if prompt treatment is not provided.

174
Q

The critical care nurse is preparing to initiate an infusion of a vasoactive medication to a patient in shock. The nurse knows that vasoactive medications are given in all forms of shock. What is the primary goal of this aspect of treatment?

A) To prevent the formation of infarcts of emboli
B) To limit stroke volume and cardiac output
C) To prevent pulmonary and peripheral edema
D) To maintain adequate mean arterial pressure

A

D) To maintain adequate mean arterial pressure

Vasoactive medications can be administered in all forms of shock to improve the patients hemodynamic stability when fluid therapy alone cannot maintain adequate MAP. Specific medications are selected to correct the particular hemodynamic alteration that is impeding cardiac output. These medications help increase the strength of myocardial contractility, regulate the heart rate, reduce myocardial resistance, and initiate vasoconstriction. They are not specifically used to prevent emboli, edema, or infarcts.

175
Q

The ICU nurse caring for a patient in shock is administering vasoactive medications as per orders. The nurse should know that vasoactive medications should be administered in what way?

A) Through a central venous line
B) By a gravity infusion IV set
C) By IV push for rapid onset of action
D) Mixed with parenteral feedings to balance osmosis

A

A) Through a central venous line

Whenever possible, vasoactive medications should be administered through a central venous line because infiltration and extravasation of some vasoactive medications can cause tissue necrosis and sloughing. An IV pump or controller must be used to ensure that the medications are delivered safely and accurately. They are never mixed with parenteral nutrition.

176
Q

The ICU nurse is caring for a patient in hypovolemic shock following a postpartum hemorrhage. For what serious complication of treatment should the nurse monitor the patient?

A) Anaphylaxis
B) Decreased oxygen consumption
C) Abdominal compartment syndrome
D) Decreased serum osmolality

A

C) Abdominal compartment syndrome

Abdominal compartment syndrome (ACS) is a serious complication that may occur when large volumes of fluid are administered. The scenario does not describe an antigenantibody reaction of any type. Decreased oxygen consumption by the body is not a concern in hypovolemic shock. With a decrease in fluids in the intravascular space, increased serum osmolality would occur.

177
Q

Sepsis is an evolving process, with neither clearly definable clinical signs and symptoms nor predictable progression. As the ICU nurse caring for a patient with sepsis, the nurse knows that tissue perfusion declines during sepsis and the patient begins to show signs of organ dysfunction. What sign would indicate to the nurse that end-organ damage may be occurring?

A) Urinary output increases
B) Skin becomes warm and dry
C) Adventitious lung sounds occur in the upper airway
D) Heart and respiratory rates are elevated

A

D) Heart and respiratory rates are elevated

As sepsis progresses, tissues become less perfused and acidotic, compensation begins to fail, and the patient begins to show signs of organ dysfunction. The cardiovascular system also begins to fail, the blood pressure does not respond to fluid resuscitation and vasoactive agents, and signs of end-organ damage are evident (e.g., renal failure, pulmonary failure, hepatic failure). As sepsis progresses to septic shock, the blood pressure drops, and the skin becomes cool, pale, and mottled. Temperature may be normal or below normal. Heart and respiratory rates remain rapid. Urine production ceases, and multiple organ dysfunction progressing to death occurs. Adventitious lung sounds occur throughout the lung fields, not just in the upper fields of the lungs.

178
Q

An 11-year-old boy has been brought to the ED by his teacher, who reports that the boy may be having a really bad allergic reaction to peanuts after trading lunches with a peer. The triage nurses rapid assessment reveals the presence of respiratory and cardiac arrest. What interventions should the nurse prioritize?

A) Establishing central venous access and beginning fluid resuscitation
B) Establishing a patent airway and beginning cardiopulmonary resuscitation
C) Establishing peripheral IV access and administering IV epinephrine
D) Performing a comprehensive assessment and initiating rapid fluid replacement

A

B) Establishing a patent airway and beginning cardiopulmonary resuscitation

If cardiac arrest and respiratory arrest are imminent or have occurred, CPR is performed. As well, a patent airway is an immediate priority. Epinephrine is not withheld pending IV access and fluid resuscitation is not a priority.

179
Q

A patient is responding poorly to interventions aimed at treating shock and appears to be transitioning to the irreversible stage of shock. What action should the intensive care nurse include during this phase of the patients care?

A) Communicate clearly and frequently with the patients family.
B) Taper down interventions slowly when the prognosis worsens.
C) Transfer the patient to a subacute unit when recovery appears unlikely.
D) Ask the patients family how they would prefer treatment to proceed.

A

A) Communicate clearly and frequently with the patients family.

As it becomes obvious that the patient is unlikely to survive, the family must be informed about the prognosis and likely outcome. Opportunities should be provided, throughout the patients care, for the family to see, touch, and talk to the patient. The onus should not be placed on the family to guide care, however. Interventions are not normally reduced gradually when they are deemed ineffective; instead, they are discontinued when they appear futile. The patient would not be transferred to a subacute unit.

180
Q

A critical care nurse is aware of the high incidence of ventilator-associated pneumonia (VAP) in patients who are being treated for shock. What intervention should be specified in the patients plan of care while the patient is ventilated?

A) Performing frequent oral care
B) Maintaining the patient in a supine position
C) Suctioning the patient every 15 minutes unless contraindicated
D) Administering prophylactic antibiotics, as ordered

A

A) Performing frequent oral care

Nursing interventions that reduce the incidence of VAP must also be implemented. These include frequent oral care, aseptic suction technique, turning, and elevating the head of the bed at least 30 degrees to prevent aspiration. Suctioning should not be excessively frequent and prophylactic antibiotics are not normally indicated.

181
Q

A patient is being treated in the ICU for neurogenic shock secondary to a spinal cord injury. Despite aggressive interventions, the patients mean arterial pressure (MAP) has fallen to 55 mm Hg. The nurse should gauge the onset of acute kidney injury by referring to what laboratory findings? Select all that apply.

A) Blood urea nitrogen (BUN) level
B) Urine specific gravity
C) Alkaline phosphatase level
D) Creatinine level
E) Serum albumin level
A

A) Blood urea nitrogen (BUN) level
B) Urine specific gravity
D) Creatinine level

Acute kidney injury (AKI) is characterized by an increase in BUN and serum creatinine levels, fluid and electrolyte shifts, acid base imbalances, and a loss of the renalhormonal regulation of BP. Urine specific gravity is also affected. Alkaline phosphatase and albumin levels are related to hepatic function.

182
Q

When caring for a patient in acute septic shock, what should the nurse anticipate?

a Infusing large amounts of IV fluids
b Administering osmotic and/or loop diuretics
c Administering IV diphenhydramine (Benadryl)
d Assisting with insertion of a ventricular assist device (VAD)

A

a Infusing large amounts of IV fluids

Septic shock is characterized by a decreased circulating blood volume. Volume expansion with the administration of IV fluids is the cornerstone of therapy. The administration of diuretics is inappropriate. VADs are useful for cardiogenic shock not septic shock. Diphenhydramine (Benadryl) may be used for anaphylactic shock but would not be helpful with septic shock.

183
Q

The critical care nurse is caring for a 55-year-old man who has a catheter in the right radial artery that is being used for continuous arterial blood pressure monitoring following his abdominal aortic aneurysm surgery. Which observation by the nurse would require an emergency intervention?

a Calculated mean arterial pressure is 74 mm Hg.
b Patient’s head of bed elevation is at 30 degrees.
c Capillary refill time in the right hand is 5 seconds.
d Pressure bag attached to the arterial line is inflated to 270 mm Hg.

A

c Capillary refill time in the right hand is 5 seconds.

Neurovascular status distal to the arterial insertion site is monitored hourly. If arterial flow is compromised, the limb will be cool and pale, with capillary refill time longer than 3 seconds. Symptoms of neurologic impairment include paresthesia, pain, or paralysis. Neurovascular impairment can result in loss of a limb and is an emergency. The pressure bag should be inflated to 300 mm Hg. Normal range for mean arterial pressure is 70 to 105 mm Hg. The backrest elevation may be up to 45 degrees unless the patient has orthostatic changes.

184
Q

The nurse is caring for a 34-year-old woman with acute decompensated heart failure who has a pulmonary artery catheter. Which assessment best indicates that the patient’s condition is improving?

a Cardiac output (CO) is 3.5 L/minute.
b Central venous pressure (CVP) is 10 mm Hg.
c Pulmonary artery wedge pressure (PAWP) is 10 mm Hg.
d Systemic vascular resistance (SVR) is 1500 dynes/sec/cm-5.

A

c Pulmonary artery wedge pressure (PAWP) is 10 mm Hg.

PAWP is the most sensitive indicator of cardiac function and fluid volume status. Normal range for PAWP is 6 to 12 mm Hg. PAWP is increased in heart failure. Normal range for CVP is 2 to 8 mm Hg. An elevated CVP indicates right-sided heart failure or volume overload. Normal cardiac output is 4 to 8 L/minute. CO is decreased in heart failure. SVR is increased in left-sided heart failure. Normal SVR is 800 to 1200 dynes/sec/cm-5.

185
Q

A 68-year-old male patient diagnosed with sepsis is orally intubated on mechanical ventilation. Which action is most important for the nurse to take?

a Use the open-suctioning technique.
b Administer morphine for discomfort.
c Limit noise and cluster care activities.
d Elevate the head of the bed 30 degrees.

A

d Elevate the head of the bed 30 degrees.

The two major complications of endotracheal intubation are unplanned extubation and aspiration. To prevent aspiration all intubated patients and patients receiving enteral feedings must have the head of the bed (HOB) elevated a minimum of 30 to 45 degrees unless medically contraindicated. Closed-suction technique is preferred over the open-suction technique because oxygenation and ventilation are maintained during suctioning, and exposure to secretions is reduced. The nurse should provide comfort measures such as morphine to relieve anxiety and pain associated with intubation. To promote rest and sleep the nurse should limit noise and cluster activities.

186
Q

A 64-year-old male patient admitted to the critical care unit for gastrointestinal hemorrhage complains of feeling tense and nervous. He appears restless with an increase in blood pressure and pulse. If the physical assessment shows no other changes, it is most important for the critical care nurse to take which action?

a Administer prescribed IV dose of lorazepam (Ativan).
b Stay with the patient and encourage expression of concerns.
c Ask a family member to remain at the bedside with the patient.
d Teach the patient how to use guided imagery to reduce anxiety.

A

b Stay with the patient and encourage expression of concerns.

Anxiety is a common problem for critically ill patients. The nurse should first stay with the patient and encourage the patient to express concerns and needs. After expression of feelings, the nurse should determine the appropriate intervention if needed (e.g., lorazepam, guided imagery, family presence) and closely monitor the patient’s hemodynamic parameters.

187
Q

Which hematologic problem most significantly increases the risks associated with pulmonary artery (PA) catheter insertion?

a Leukocytosis
b Hypovolemia
c Hemolytic anemia
d Thrombocytopenia

A

d Thrombocytopenia

PA catheter insertion carries a significant risk of bleeding, a fact that is exacerbated when the patient has low levels of platelets. Leukocytosis, hypovolemia, and anemia are less likely to directly increase the risks associated with PA insertion.

188
Q

The patient has developed cardiogenic shock after a left anterior descending myocardial infection. Which circulatory-assist device should the nurse expect to use for this patient?

a Cardiopulmonary bypass
b Impedance cardiography (ICG)
c Intraaortic balloon pump (IABP)
d Central venous pressure (CVP) measurement

A

c Intraaortic balloon pump (IABP)

The most commonly used mechanical circulatory-assist device is the intraaortic balloon pump (IABP), and it is used to decrease ventricular workload, increase myocardial perfusion, and augment circulation. Cardiopulmonary bypass provides circulation during open heart surgery. It is not used as an assist device after surgery. ICG is a noninvasive method to obtain cardiac output and assess thoracic fluid status. CVP measurement is an invasive measurement of right ventricular preload and reflects fluid volume problems.

189
Q

A 70-year-old patient in the ICU has become agitated and inattentive since his heart surgery. The nurse knows that this ICU psychosis frequently occurs in individuals with pre-existing dementia, history of alcohol abuse, and severe disease. What interventions should the nurse provide this patient to improve the patient’s cognition (select all that apply)?

a Improve oxygenation.
b Provide a small amount of beer.
c Have the family stay with the patient.
d Enable the patient to sleep on a schedule with dim lights.
e Decrease sensory overload by conversing away from patient’s room.

A

a Improve oxygenation.
d Enable the patient to sleep on a schedule with dim lights.
e Decrease sensory overload by conversing away from patient’s room.

ICU psychosis is from delirium in most ICU patients. Improving oxygenation, enabling the patient to sleep, and decreasing sensory overload along with orientation is all helpful in improving the patient’s cognition. The beer may or may not be allowed for this patient, and the nurse should not assume that it will help. Having a family member stay with the patient to reorient the patient is helpful, but the family group may increase sensory overload with conversations not involving the patient.

190
Q

To establish hemodynamic monitoring for a patient, the nurse zeroes the:

a. cardiac output monitoring system to the level of the left ventricle
b. pressure monitoring system to the level of the catheter tip located in the patient
c. pressure monitoring system to the level of the atrium, identified as the phlebostatic axis
d. pressure monitoring system to the level of the atrium, identified as the midclavicular line

A

c. pressure monitoring system to the level of the atrium, identified as the phlebostatic axis

191
Q

The hemodynamic changes the nurse expects to find after successful initiation of IABP therapy in a patient with cardiogenic shock include (select all that apply):

a. decreased SV
b. decreased SVR
c. decreased PAWP
d. increased diastolic BP
e. a decreased myocardial oxygen consumption

A

b. decreased SVR
c. decreased PAWP
d. increased diastolic BP
e. a decreased myocardial oxygen consumption